Thread Rating:

Poll

2 votes (16.66%)
3 votes (25%)
2 votes (16.66%)
3 votes (25%)
No votes (0%)
1 vote (8.33%)
1 vote (8.33%)
1 vote (8.33%)
4 votes (33.33%)
2 votes (16.66%)

12 members have voted

Wizard
Administrator
Wizard
  • Threads: 1493
  • Posts: 26485
Joined: Oct 14, 2009
May 20th, 2019 at 3:04:04 PM permalink
I am doing a software review of a company that offers prop bets on the sum of the balls in keno. As a reminder, in keno the game draws 20 numbers from a 80 balls numbered 1 to 80. The problem at hand is finding a probability of any given total without using a simulation.

I do know simulations are a perfectly valid way of analyzing casino games, but I also feel more satisfied finding an exact answer. At first I wrote a simple program to loop through 3,535,316,142,212,180,000 possible ways to choose 20 out of 80 balls. Needless to say, it would have taken centuries to cycle through them all. I've also toyed with shortcut code, but I fear I'll die before the program is done running.

I've also just counted combinations by hand, hoping to find a pattern (maybe some variant of Fibonaci), but so far have not found anything. So, at this point I open it up to the forum. You do not have to put replies in spoiler tags and may refer to any outside sources. This isn't the usual math puzzle thread.

The question for the poll is what are your thoughts on it?

In this spoiler tag are the results of my simulation. I excluded extreme totals where the probability was less than 1 in a million. It's a big table, so I won't clutter up the thread with it, unless you want to see it. The third column is the expected count per million games.


Total Count Exp. per Mil.
441 3,435 1
442 3,744 1
443 3,838 1
444 3,969 1
445 4,323 1
446 4,534 1
447 4,739 1
448 5,044 1
449 5,321 1
450 5,545 1
451 5,796 1
452 6,293 1
453 6,611 1
454 6,999 1
455 7,282 1
456 7,860 1
457 7,940 1
458 8,452 1
459 9,068 1
460 9,480 1
461 10,009 2
462 10,574 2
463 11,129 2
464 11,514 2
465 12,115 2
466 12,753 2
467 13,486 2
468 14,189 2
469 14,828 2
470 15,359 2
471 16,377 3
472 17,251 3
473 18,045 3
474 19,033 3
475 19,760 3
476 20,672 3
477 21,744 3
478 22,939 4
479 24,007 4
480 25,196 4
481 26,055 4
482 27,080 4
483 28,661 4
484 29,927 5
485 31,384 5
486 32,913 5
487 34,522 5
488 36,028 6
489 37,678 6
490 39,692 6
491 41,261 6
492 43,166 7
493 45,217 7
494 47,199 7
495 48,906 8
496 51,285 8
497 53,787 8
498 56,207 9
499 58,636 9
500 61,251 9
501 63,618 10
502 66,347 10
503 69,196 11
504 72,198 11
505 75,956 12
506 78,432 12
507 81,814 13
508 85,550 13
509 89,007 14
510 92,998 14
511 96,468 15
512 101,191 16
513 104,498 16
514 109,120 17
515 113,433 17
516 118,328 18
517 122,687 19
518 127,485 20
519 134,015 21
520 138,739 21
521 144,682 22
522 149,379 23
523 155,583 24
524 161,377 25
525 167,986 26
526 174,464 27
527 181,674 28
528 188,399 29
529 196,313 30
530 202,955 31
531 211,699 32
532 217,869 33
533 228,639 35
534 235,939 36
535 245,385 38
536 253,054 39
537 262,573 40
538 272,414 42
539 282,960 43
540 293,067 45
541 303,583 47
542 315,744 48
543 325,370 50
544 339,232 52
545 350,113 54
546 362,701 56
547 376,191 58
548 388,543 60
549 402,589 62
550 416,646 64
551 431,632 66
552 446,715 69
553 461,115 71
554 476,083 73
555 493,312 76
556 509,761 78
557 527,002 81
558 545,283 84
559 563,285 86
560 579,537 89
561 600,939 92
562 620,205 95
563 640,774 98
564 663,018 102
565 682,220 105
566 705,101 108
567 728,213 112
568 751,145 115
569 773,003 119
570 800,365 123
571 822,985 126
572 849,868 130
573 877,063 135
574 903,857 139
575 933,334 143
576 960,447 147
577 990,405 152
578 1,021,121 157
579 1,051,043 161
580 1,081,462 166
581 1,115,569 171
582 1,149,091 176
583 1,182,506 181
584 1,218,923 187
585 1,252,892 192
586 1,289,475 198
587 1,326,081 203
588 1,365,000 209
589 1,404,936 215
590 1,447,113 222
591 1,486,588 228
592 1,527,549 234
593 1,572,339 241
594 1,612,520 247
595 1,659,021 254
596 1,705,695 262
597 1,754,146 269
598 1,801,657 276
599 1,850,683 284
600 1,899,207 291
601 1,953,142 300
602 2,005,515 308
603 2,057,840 316
604 2,111,828 324
605 2,165,195 332
606 2,222,992 341
607 2,283,008 350
608 2,339,728 359
609 2,399,839 368
610 2,462,058 378
611 2,522,864 387
612 2,590,626 397
613 2,653,379 407
614 2,720,129 417
615 2,789,066 428
616 2,857,378 438
617 2,927,708 449
618 3,005,437 461
619 3,076,651 472
620 3,145,799 482
621 3,223,678 494
622 3,298,823 506
623 3,375,968 518
624 3,458,561 530
625 3,540,982 543
626 3,621,286 555
627 3,706,336 568
628 3,793,415 582
629 3,879,513 595
630 3,968,574 609
631 4,059,927 623
632 4,147,717 636
633 4,240,094 650
634 4,333,209 665
635 4,427,877 679
636 4,526,070 694
637 4,627,644 710
638 4,727,946 725
639 4,830,073 741
640 4,930,411 756
641 5,035,769 772
642 5,144,626 789
643 5,249,616 805
644 5,363,903 823
645 5,469,158 839
646 5,584,422 857
647 5,698,184 874
648 5,821,229 893
649 5,931,005 910
650 6,053,589 928
651 6,170,477 946
652 6,296,285 966
653 6,419,240 985
654 6,547,043 1,004
655 6,667,303 1,023
656 6,796,655 1,042
657 6,927,675 1,063
658 7,055,827 1,082
659 7,189,622 1,103
660 7,326,894 1,124
661 7,456,552 1,144
662 7,595,410 1,165
663 7,741,620 1,187
664 7,878,448 1,208
665 8,024,240 1,231
666 8,164,932 1,252
667 8,310,177 1,275
668 8,456,983 1,297
669 8,607,591 1,320
670 8,762,016 1,344
671 8,900,482 1,365
672 9,061,785 1,390
673 9,209,800 1,413
674 9,370,622 1,437
675 9,530,537 1,462
676 9,682,898 1,485
677 9,840,473 1,509
678 10,002,377 1,534
679 10,166,739 1,559
680 10,332,192 1,585
681 10,489,367 1,609
682 10,656,721 1,634
683 10,833,480 1,662
684 10,997,379 1,687
685 11,164,607 1,712
686 11,334,202 1,738
687 11,511,102 1,766
688 11,683,505 1,792
689 11,854,861 1,818
690 12,026,976 1,845
691 12,206,568 1,872
692 12,378,607 1,899
693 12,560,691 1,926
694 12,737,586 1,954
695 12,921,619 1,982
696 13,099,750 2,009
697 13,281,463 2,037
698 13,464,349 2,065
699 13,659,987 2,095
700 13,831,868 2,121
701 14,015,126 2,150
702 14,205,072 2,179
703 14,392,695 2,207
704 14,581,750 2,236
705 14,757,007 2,263
706 14,956,306 2,294
707 15,144,221 2,323
708 15,330,541 2,351
709 15,523,000 2,381
710 15,707,557 2,409
711 15,901,310 2,439
712 16,090,644 2,468
713 16,277,131 2,496
714 16,474,557 2,527
715 16,657,263 2,555
716 16,859,581 2,586
717 17,044,543 2,614
718 17,241,315 2,644
719 17,431,764 2,674
720 17,621,957 2,703
721 17,820,359 2,733
722 18,007,127 2,762
723 18,196,152 2,791
724 18,390,894 2,821
725 18,581,770 2,850
726 18,776,054 2,880
727 18,952,584 2,907
728 19,150,208 2,937
729 19,340,928 2,966
730 19,530,508 2,995
731 19,711,150 3,023
732 19,909,926 3,054
733 20,090,359 3,081
734 20,276,212 3,110
735 20,458,780 3,138
736 20,645,075 3,166
737 20,826,094 3,194
738 21,010,505 3,222
739 21,184,684 3,249
740 21,374,281 3,278
741 21,542,413 3,304
742 21,729,944 3,333
743 21,905,866 3,360
744 22,084,327 3,387
745 22,253,227 3,413
746 22,419,160 3,439
747 22,594,055 3,465
748 22,775,467 3,493
749 22,936,395 3,518
750 23,101,841 3,543
751 23,278,659 3,570
752 23,442,393 3,595
753 23,594,853 3,619
754 23,764,936 3,645
755 23,915,848 3,668
756 24,072,575 3,692
757 24,219,496 3,715
758 24,382,858 3,740
759 24,537,006 3,763
760 24,683,616 3,786
761 24,822,836 3,807
762 24,986,511 3,832
763 25,116,433 3,852
764 25,255,618 3,874
765 25,398,888 3,896
766 25,526,481 3,915
767 25,672,383 3,937
768 25,791,251 3,956
769 25,915,025 3,975
770 26,050,458 3,995
771 26,178,986 4,015
772 26,296,271 4,033
773 26,413,333 4,051
774 26,528,066 4,069
775 26,636,170 4,085
776 26,748,652 4,103
777 26,857,717 4,119
778 26,969,571 4,136
779 27,055,593 4,150
780 27,167,435 4,167
781 27,261,216 4,181
782 27,355,200 4,196
783 27,453,032 4,211
784 27,520,647 4,221
785 27,609,338 4,235
786 27,683,154 4,246
787 27,783,913 4,261
788 27,851,963 4,272
789 27,920,350 4,282
790 27,985,234 4,292
791 28,042,985 4,301
792 28,118,461 4,313
793 28,177,419 4,322
794 28,230,306 4,330
795 28,279,969 4,337
796 28,330,982 4,345
797 28,376,165 4,352
798 28,405,742 4,357
799 28,457,425 4,365
800 28,508,733 4,373
801 28,522,647 4,375
802 28,548,374 4,379
803 28,585,242 4,384
804 28,592,528 4,385
805 28,623,406 4,390
806 28,642,581 4,393
807 28,643,977 4,393
808 28,650,200 4,394
809 28,662,962 4,396
810 28,663,683 4,396
811 28,666,272 4,397
812 28,659,446 4,396
813 28,642,707 4,393
814 28,636,339 4,392
815 28,623,072 4,390
816 28,602,453 4,387
817 28,577,793 4,383
818 28,558,620 4,380
819 28,525,239 4,375
820 28,484,774 4,369
821 28,454,276 4,364
822 28,424,946 4,360
823 28,369,113 4,351
824 28,336,596 4,346
825 28,280,514 4,338
826 28,229,969 4,330
827 28,179,004 4,322
828 28,115,128 4,312
829 28,050,436 4,302
830 27,978,178 4,291
831 27,923,254 4,283
832 27,851,480 4,272
833 27,775,285 4,260
834 27,696,831 4,248
835 27,614,904 4,235
836 27,523,482 4,221
837 27,446,944 4,210
838 27,350,877 4,195
839 27,259,610 4,181
840 27,166,593 4,167
841 27,077,387 4,153
842 26,959,292 4,135
843 26,857,548 4,119
844 26,737,880 4,101
845 26,644,740 4,087
846 26,530,087 4,069
847 26,409,425 4,051
848 26,293,674 4,033
849 26,179,346 4,015
850 26,043,665 3,994
851 25,924,860 3,976
852 25,786,873 3,955
853 25,667,874 3,937
854 25,534,938 3,916
855 25,394,716 3,895
856 25,266,400 3,875
857 25,117,582 3,852
858 24,974,923 3,831
859 24,827,566 3,808
860 24,681,658 3,786
861 24,530,307 3,762
862 24,380,059 3,739
863 24,229,392 3,716
864 24,067,458 3,691
865 23,916,986 3,668
866 23,763,351 3,645
867 23,595,872 3,619
868 23,435,026 3,594
869 23,272,670 3,569
870 23,106,061 3,544
871 22,943,980 3,519
872 22,768,413 3,492
873 22,605,841 3,467
874 22,430,941 3,440
875 22,250,856 3,413
876 22,084,770 3,387
877 21,909,607 3,360
878 21,734,749 3,334
879 21,552,728 3,306
880 21,368,700 3,277
881 21,193,387 3,251
882 20,999,640 3,221
883 20,822,422 3,194
884 20,643,153 3,166
885 20,462,048 3,138
886 20,272,437 3,109
887 20,090,583 3,081
888 19,908,019 3,053
889 19,718,060 3,024
890 19,525,110 2,995
891 19,333,451 2,965
892 19,148,662 2,937
893 18,959,539 2,908
894 18,764,734 2,878
895 18,579,508 2,850
896 18,393,824 2,821
897 18,199,883 2,791
898 18,014,782 2,763
899 17,820,332 2,733
900 17,622,591 2,703
901 17,431,435 2,674
902 17,237,539 2,644
903 17,047,999 2,615
904 16,851,334 2,585
905 16,671,357 2,557
906 16,464,875 2,525
907 16,285,487 2,498
908 16,088,821 2,468
909 15,901,591 2,439
910 15,712,769 2,410
911 15,523,156 2,381
912 15,328,998 2,351
913 15,142,681 2,322
914 14,948,183 2,293
915 14,768,973 2,265
916 14,581,616 2,236
917 14,393,031 2,208
918 14,201,510 2,178
919 14,022,741 2,151
920 13,840,535 2,123
921 13,637,420 2,092
922 13,469,777 2,066
923 13,279,699 2,037
924 13,103,752 2,010
925 12,918,928 1,981
926 12,737,099 1,954
927 12,565,003 1,927
928 12,382,420 1,899
929 12,202,822 1,872
930 12,034,624 1,846
931 11,852,584 1,818
932 11,679,705 1,791
933 11,506,740 1,765
934 11,332,814 1,738
935 11,171,123 1,713
936 10,989,047 1,685
937 10,827,757 1,661
938 10,656,739 1,634
939 10,488,145 1,609
940 10,333,650 1,585
941 10,166,315 1,559
942 9,999,714 1,534
943 9,842,566 1,510
944 9,685,425 1,485
945 9,524,252 1,461
946 9,371,438 1,437
947 9,213,035 1,413
948 9,063,459 1,390
949 8,903,743 1,366
950 8,760,545 1,344
951 8,607,733 1,320
952 8,454,229 1,297
953 8,309,658 1,274
954 8,165,969 1,252
955 8,024,407 1,231
956 7,884,097 1,209
957 7,741,275 1,187
958 7,602,600 1,166
959 7,467,518 1,145
960 7,326,924 1,124
961 7,186,025 1,102
962 7,059,347 1,083
963 6,925,203 1,062
964 6,797,473 1,043
965 6,671,659 1,023
966 6,542,112 1,003
967 6,417,141 984
968 6,296,293 966
969 6,172,646 947
970 6,054,444 929
971 5,934,001 910
972 5,816,099 892
973 5,696,308 874
974 5,585,518 857
975 5,470,602 839
976 5,363,579 823
977 5,250,182 805
978 5,143,910 789
979 5,036,934 773
980 4,934,663 757
981 4,829,209 741
982 4,727,906 725
983 4,629,768 710
984 4,524,975 694
985 4,427,515 679
986 4,335,792 665
987 4,240,216 650
988 4,147,607 636
989 4,055,769 622
990 3,965,357 608
991 3,879,116 595
992 3,793,847 582
993 3,708,386 569
994 3,620,849 555
995 3,539,124 543
996 3,456,841 530
997 3,378,144 518
998 3,299,675 506
999 3,223,531 494
1,000 3,149,948 483
1,001 3,073,548 471
1,002 2,999,209 460
1,003 2,930,573 449
1,004 2,860,976 439
1,005 2,791,039 428
1,006 2,721,344 417
1,007 2,656,566 407
1,008 2,589,961 397
1,009 2,524,701 387
1,010 2,462,746 378
1,011 2,405,928 369
1,012 2,342,078 359
1,013 2,283,811 350
1,014 2,222,728 341
1,015 2,163,391 332
1,016 2,111,514 324
1,017 2,058,080 316
1,018 2,002,275 307
1,019 1,952,564 299
1,020 1,899,055 291
1,021 1,852,096 284
1,022 1,800,065 276
1,023 1,752,832 269
1,024 1,705,132 262
1,025 1,657,875 254
1,026 1,617,609 248
1,027 1,569,189 241
1,028 1,528,429 234
1,029 1,485,557 228
1,030 1,445,678 222
1,031 1,402,081 215
1,032 1,364,455 209
1,033 1,327,805 204
1,034 1,289,037 198
1,035 1,254,056 192
1,036 1,216,909 187
1,037 1,183,935 182
1,038 1,148,374 176
1,039 1,113,641 171
1,040 1,081,610 166
1,041 1,049,225 161
1,042 1,019,412 156
1,043 990,011 152
1,044 960,185 147
1,045 932,152 143
1,046 903,127 139
1,047 878,067 135
1,048 849,403 130
1,049 825,986 127
1,050 800,391 123
1,051 774,569 119
1,052 750,355 115
1,053 727,303 112
1,054 705,731 108
1,055 681,719 105
1,056 662,209 102
1,057 642,753 99
1,058 620,171 95
1,059 601,326 92
1,060 581,228 89
1,061 563,149 86
1,062 543,921 83
1,063 527,559 81
1,064 510,221 78
1,065 492,829 76
1,066 476,649 73
1,067 461,857 71
1,068 446,006 68
1,069 431,170 66
1,070 416,983 64
1,071 403,009 62
1,072 388,740 60
1,073 375,730 58
1,074 362,744 56
1,075 349,799 54
1,076 337,515 52
1,077 325,578 50
1,078 315,035 48
1,079 304,850 47
1,080 293,324 45
1,081 282,717 43
1,082 272,825 42
1,083 262,148 40
1,084 253,864 39
1,085 244,561 38
1,086 236,658 36
1,087 227,144 35
1,088 219,325 34
1,089 211,325 32
1,090 203,347 31
1,091 196,097 30
1,092 188,406 29
1,093 182,111 28
1,094 174,976 27
1,095 167,875 26
1,096 161,607 25
1,097 156,189 24
1,098 149,548 23
1,099 144,428 22
1,100 138,467 21
1,101 133,467 20
1,102 128,131 20
1,103 123,875 19
1,104 118,380 18
1,105 113,401 17
1,106 109,324 17
1,107 105,067 16
1,108 100,365 15
1,109 96,211 15
1,110 92,518 14
1,111 89,418 14
1,112 85,559 13
1,113 82,129 13
1,114 78,608 12
1,115 75,594 12
1,116 72,488 11
1,117 69,493 11
1,118 65,888 10
1,119 63,739 10
1,120 60,695 9
1,121 58,616 9
1,122 56,145 9
1,123 53,379 8
1,124 51,672 8
1,125 49,369 8
1,126 46,770 7
1,127 44,880 7
1,128 43,118 7
1,129 41,257 6
1,130 39,401 6
1,131 37,876 6
1,132 35,977 6
1,133 34,743 5
1,134 32,984 5
1,135 31,269 5
1,136 29,765 5
1,137 28,461 4
1,138 27,738 4
1,139 26,374 4
1,140 24,998 4
1,141 23,762 4
1,142 22,931 4
1,143 21,563 3
1,144 20,568 3
1,145 19,825 3
1,146 18,735 3
1,147 17,862 3
1,148 17,258 3
1,149 16,271 2
1,150 15,606 2
1,151 14,719 2
1,152 14,241 2
1,153 13,258 2
1,154 12,837 2
1,155 11,988 2
1,156 11,648 2
1,157 10,916 2
1,158 10,411 2
1,159 9,810 2
1,160 9,449 1
1,161 8,923 1
1,162 8,499 1
1,163 8,170 1
1,164 7,715 1
1,165 7,342 1
1,166 6,914 1
1,167 6,632 1
1,168 6,243 1
1,169 5,866 1
1,170 5,757 1
1,171 5,388 1
1,172 4,976 1
1,173 4,800 1
1,174 4,605 1
1,175 4,282 1
1,176 3,958 1
1,177 3,703 1
1,178 3,722 1
1,179 3,478 1
Total 6,520,000,000 1,000,000

"For with much wisdom comes much sorrow." -- Ecclesiastes 1:18 (NIV)
7craps
7craps
  • Threads: 18
  • Posts: 1977
Joined: Jan 23, 2010
May 20th, 2019 at 3:21:59 PM permalink
Quote: Wizard

I am doing a software review of a company that offers prop bets on the sum of the balls in keno. As a reminder, in keno the game draws 20 numbers from a 80 balls numbered 1 to 80.

The problem at hand is finding a probability of any given total without using a simulation.

Been done as is very simple
uses generating function (I know that much)

"or in Pari Gp (free)
prod(i=1, 80, 1 + x*y^i);"

one needs only the y values for x^20 and I have not verified this method. I think this was found online.

https://wizardofvegas.com/forum/questions-and-answers/gambling/10526-what-is-keno-exact-scores-house-edge/3/#post652407

the results exactly for all the sums can be found here
that page has yet to be cleaned up. text file and Excel

https://sites.google.com/view/krapstuff/keno
winsome johnny (not Win some johnny)
Wizard
Administrator
Wizard
  • Threads: 1493
  • Posts: 26485
Joined: Oct 14, 2009
May 20th, 2019 at 3:38:56 PM permalink
Interesting. I wish Sally were still around to convert that to more plain simple English. Can anyone else help? Here are the combinations from Sally's source. Again, putting table in spoiler tags because it's so big. I can't post the whole table because there is a maximum size per post.


Total Combinations
210 1
211 1
212 2
213 3
214 5
215 7
216 11
217 15
218 22
219 30
220 42
221 56
222 77
223 101
224 135
225 176
226 231
227 297
228 385
229 490
230 627
231 791
232 1,000
233 1,251
234 1,568
235 1,946
236 2,417
237 2,980
238 3,673
239 4,498
240 5,507
241 6,703
242 8,154
243 9,871
244 11,937
245 14,375
246 17,293
247 20,722
248 24,803
249 29,588
250 35,251
251 41,869
252 49,668
253 58,754
254 69,414
255 81,801
256 96,271
257 113,039
258 132,559
259 155,112
260 181,274
261 211,428
262 246,288
263 286,364
264 332,557
265 385,528
266 446,405
267 516,054
268 595,872
269 686,983
270 791,131
271 909,741
272 1,044,984
273 1,198,689
274 1,373,524
275 1,571,812
276 1,796,855
277 2,051,569
278 2,340,024
279 2,665,885
280 3,034,135
281 3,449,359
282 3,917,670
283 4,444,748
284 5,038,070
285 5,704,686
286 6,453,684
287 7,293,767
288 8,236,001
289 9,291,060
290 10,472,375
291 11,793,028
292 13,269,250
293 14,917,024
294 16,755,957
295 18,805,464
296 21,089,176
297 23,630,664
298 26,458,297
299 29,600,587
300 33,091,578
301 36,965,620
302 41,263,462
303 46,026,431
304 51,303,148
305 57,143,201
306 63,604,540
307 70,746,444
308 78,637,842
309 87,349,501
310 96,963,169
311 107,563,103
312 119,246,146
313 132,112,328
314 146,276,124
315 161,856,093
316 178,987,225
317 197,809,729
318 218,482,569
319 241,171,101
320 266,062,168
321 293,350,487
322 323,255,164
323 356,005,000
324 391,856,581
325 431,078,138
326 473,969,587
327 520,844,871
328 572,053,851
329 627,963,266
330 688,980,643
331 755,533,468
332 828,095,629
333 907,164,667
334 993,290,571
335 1,087,051,347
336 1,189,084,411
337 1,300,059,954
338 1,420,715,489
339 1,551,826,889
340 1,694,245,990
341 1,848,869,496
342 2,016,679,881
343 2,198,711,618
344 2,396,096,082
345 2,610,024,936
346 2,841,798,846
347 3,092,788,314
348 3,364,486,581
349 3,658,467,191
350 3,976,441,876
351 4,320,214,692
352 4,691,744,778
353 5,093,097,335
354 5,526,511,602
355 5,994,347,965
356 6,499,162,145
357 7,043,648,186
358 7,630,718,671
359 8,263,443,950
360 8,945,138,936
361 9,679,297,643
362 10,469,687,646
363 11,320,279,729
364 12,235,349,889
365 13,219,404,469
366 14,277,290,236
367 15,414,114,032
368 16,635,362,300
369 17,946,814,901
370 19,354,673,930
371 20,865,472,184
372 22,486,212,019
373 24,224,267,278
374 26,087,533,643
375 28,084,323,773
376 30,223,529,082
377 32,514,508,546
378 34,967,262,684
379 37,592,314,751
380 40,400,898,786
381 43,404,832,757
382 46,616,720,518
383 50,049,817,550
384 53,718,247,833
385 57,636,860,670
386 61,821,464,449
387 66,288,674,187
388 71,056,162,189
389 76,142,496,873
390 81,567,411,419
391 87,351,632,249
392 93,517,168,095
393 100,087,127,594
394 107,086,028,628
395 114,539,605,867
396 122,475,141,756
397 130,921,262,045
398 139,908,291,043
399 149,468,034,620
400 159,634,159,836
401 170,441,966,142
402 181,928,790,813
403 194,133,766,301
404 207,098,254,570
405 220,865,589,927
406 235,481,542,190
407 250,994,045,973
408 267,453,694,512
409 284,913,452,879
410 303,429,185,857
411 323,059,354,425
412 343,865,577,844
413 365,912,314,318
414 389,267,459,099
415 414,002,006,749
416 440,190,689,221
417 467,911,618,783
418 497,246,966,079
419 528,282,584,833
420 561,108,732,221
421 595,819,672,189
422 632,514,442,353
423 671,296,435,063
424 712,274,211,009
425 755,561,059,108
426 801,275,859,118
427 849,542,617,012
428 900,491,381,529
429 954,257,753,733
430 1,010,983,857,544
431 1,070,817,824,773
432 1,133,914,822,434
433 1,200,436,509,276
434 1,270,552,124,993
435 1,344,437,917,067
436 1,422,278,292,091
437 1,504,265,214,046
438 1,590,599,420,536
439 1,681,489,788,194
440 1,777,154,619,819
441 1,877,820,975,168
442 1,983,726,028,916
443 2,095,116,368,246
444 2,212,249,424,868
445 2,335,392,734,382
446 2,464,825,448,513
447 2,600,837,554,495
448 2,743,731,467,669
449 2,893,821,211,946
450 3,051,434,096,031
451 3,216,909,849,508
452 3,390,602,389,686
453 3,572,878,910,940
454 3,764,121,741,811
455 3,964,727,389,027
456 4,175,108,488,550
457 4,395,692,797,816
458 4,626,925,247,931
459 4,869,266,881,582
460 5,123,197,006,289
461 5,389,212,078,927
462 5,667,827,963,566
463 5,959,578,755,669
464 6,265,019,152,487
465 6,584,723,213,704
466 6,919,286,843,610
467 7,269,326,489,361
468 7,635,481,738,958
469 8,018,413,948,848
470 8,418,808,965,914
471 8,837,375,680,898
472 9,274,848,873,719
473 9,731,987,693,363
474 10,209,578,630,140
475 10,708,433,913,262
476 11,229,394,619,008
477 11,773,328,981,483
478 12,341,135,635,235
479 12,933,741,839,899
480 13,552,106,861,442
481 14,197,220,100,303
482 14,870,104,620,270
483 15,571,815,175,325
484 16,303,441,884,554
485 17,066,108,157,849
486 17,860,974,520,443
487 18,689,236,426,307
488 19,552,128,242,181
489 20,450,920,942,555
490 21,386,926,250,737
491 22,361,494,215,565
492 23,376,017,513,498
493 24,431,928,894,154
494 25,530,705,653,070
495 26,673,866,939,713
496 27,862,978,398,196
497 29,099,649,337,087
498 30,385,537,541,434
499 31,722,346,290,416
500 33,111,829,351,342
501 34,555,787,837,266
502 36,056,075,379,039
503 37,614,594,822,327
504 39,233,303,581,272
505 40,914,210,159,126
506 42,659,379,693,854
507 44,470,930,294,083
508 46,351,038,772,509
509 48,301,936,795,528
510 50,325,916,807,381
511 52,425,327,976,532
512 54,602,582,321,801
513 56,860,150,445,667
514 59,200,567,856,702
515 61,626,430,488,492
516 64,140,401,221,500
517 66,745,205,168,528
518 69,443,636,407,019
519 72,238,553,020,681
520 75,132,884,036,461
521 78,129,624,219,915
522 81,231,841,219,122
523 84,442,670,093,739
524 87,765,320,677,722
525 91,203,071,830,294
526 94,759,279,010,024
527 98,437,368,245,632
528 102,240,843,924,128
529 106,173,282,459,017
530 110,238,340,304,328
531 114,439,747,308,027
532 118,781,314,944,316
533 123,266,929,349,329
534 127,900,559,773,363
535 132,686,251,275,763
536 137,628,133,409,981
537 142,730,412,563,985
538 147,997,380,868,998
539 153,433,408,182,971
540 159,042,951,225,485
541 164,830,545,178,892
542 170,800,813,062,207
543 176,958,456,936,318
544 183,308,267,507,535
545 189,855,114,934,011
546 196,603,958,660,512
547 203,559,837,800,120
548 210,727,881,214,639
549 218,113,297,456,569
550 225,721,385,084,601
551 233,557,522,163,930
552 241,627,176,819,381
553 249,935,896,266,269
554 258,489,317,615,563
555 267,293,156,421,037
556 276,353,217,726,061
557 285,675,384,124,940
558 295,265,627,053,394
559 305,129,994,336,046
560 315,274,621,737,283
561 325,705,719,979,621
562 336,429,586,543,371
563 347,452,592,155,562
564 358,781,192,841,696
565 370,421,915,855,964
566 382,381,372,002,154
567 394,666,240,991,089
568 407,283,284,019,903
569 420,239,328,556,562
570 433,541,281,180,913
571 447,196,111,767,398
572 461,210,866,607,609
573 475,592,651,976,889
574 490,348,647,526,558
575 505,486,089,237,058
576 521,012,283,085,036
577 536,934,587,353,256
578 553,260,426,593,322
579 569,997,273,279,789
580 587,152,662,058,258
581 604,734,170,747,184
582 622,749,434,876,858
583 641,206,128,010,489
584 660,111,976,597,920
585 679,474,739,604,064
586 699,302,223,666,421
587 719,602,262,037,791
588 740,382,730,055,707
589 761,651,523,370,470
590 783,416,573,752,126
591 805,685,826,594,439
592 828,467,257,050,350
593 851,768,846,820,577
594 875,598,600,626,154
595 899,964,522,255,079
596 924,874,631,401,209
597 950,336,938,960,611
598 976,359,464,227,661
599 1,002,950,209,453,770
600 1,030,117,177,412,690
601 1,057,868,345,195,040
602 1,086,211,682,173,950
603 1,115,155,123,030,640
604 1,144,706,586,113,290
605 1,174,873,945,708,900
606 1,205,665,050,808,210
607 1,237,087,696,599,980
608 1,269,149,643,677,450
609 1,301,858,588,751,630
610 1,335,222,184,292,440
611 1,369,248,008,480,890
612 1,403,943,585,301,200
613 1,439,316,353,708,700
614 1,475,373,688,210,190
615 1,512,122,867,248,400
616 1,549,571,094,266,050
617 1,587,725,465,329,520
618 1,626,592,990,693,830
619 1,666,180,561,645,290
620 1,706,494,972,609,520
621 1,747,542,887,215,360
622 1,789,330,860,941,280
623 1,831,865,306,428,570
624 1,875,152,516,686,540
625 1,919,198,629,635,730
626 1,964,009,651,915,660
627 2,009,591,422,664,540
628 2,055,949,637,927,470
629 2,103,089,813,703,010
630 2,151,017,310,973,090
631 2,199,737,298,000,370
632 2,249,254,776,027,770
633 2,299,574,540,834,780
634 2,350,701,209,103,640
635 2,402,639,179,270,460
636 2,455,392,658,582,210
637 2,508,965,623,226,930
638 2,563,361,846,130,740
639 2,618,584,856,379,090
640 2,674,637,967,752,940
641 2,731,524,237,478,950
642 2,789,246,495,528,880
643 2,847,807,302,685,920
644 2,907,208,980,659,850
645 2,967,453,569,482,800
646 3,028,542,858,437,460
647 3,090,478,342,825,040
648 3,153,261,255,732,850
649 3,216,892,524,160,980
650 3,281,372,801,682,290
651 3,346,702,423,944,310
652 3,412,881,442,219,840
653 3,479,909,578,328,190
654 3,547,786,259,101,070
655 3,616,510,570,712,250
656 3,686,081,294,113,980
657 3,756,496,858,791,470
658 3,827,755,379,165,350
659 3,899,854,607,818,590
660 3,972,791,972,891,010
661 4,046,564,530,766,010
662 4,121,169,004,511,070
663 4,196,601,736,043,210
664 4,272,858,725,610,060
665 4,349,935,583,482,260
666 4,427,827,570,497,980
667 4,506,529,549,270,560
668 4,586,036,025,851,500
669 4,666,341,100,470,600
670 4,747,438,510,307,980
671 4,829,321,579,817,140
672 4,911,983,264,628,410
673 4,995,416,101,440,400
674 5,079,612,253,106,590
675 5,164,563,458,113,970
676 5,250,261,076,840,900
677 5,336,696,040,671,980
678 5,423,858,899,445,150
679 5,511,739,770,187,560
680 5,600,328,385,803,350
681 5,689,614,043,446,160
682 5,779,585,654,429,600
683 5,870,231,692,286,640
684 5,961,540,243,918,360
685 6,053,498,957,320,960
686 6,146,095,094,019,120
687 6,239,315,476,476,150
688 6,333,146,541,790,360
689 6,427,574,288,835,360
690 6,522,584,333,229,140
691 6,618,161,854,183,300
692 6,714,291,650,789,160
693 6,810,958,088,586,950
694 6,908,145,157,139,450
695 7,005,836,416,368,480
696 7,104,015,055,421,620
697 7,202,663,838,749,380
698 7,301,765,166,303,070
699 7,401,301,019,361,720
700 7,501,253,022,027,260
701 7,601,602,386,843,980
702 7,702,329,977,589,080
703 7,803,416,254,655,220
704 7,904,841,339,170,870
705 8,006,584,958,150,180
706 8,108,626,509,899,880
707 8,210,945,008,978,190
708 8,313,519,152,881,930
709 8,416,327,266,778,080
710 8,519,347,371,498,560
711 8,622,557,128,046,050
712 8,725,933,906,847,840
713 8,829,454,732,072,480
714 8,933,096,352,127,630
715 9,036,835,183,748,460
716 9,140,647,383,762,520
717 9,244,508,792,945,670
718 9,348,395,008,996,000
719 9,452,281,330,191,890
720 9,556,142,829,555,640
721 9,659,954,298,267,300
722 9,763,690,321,033,400
723 9,867,325,219,249,980
724 9,970,833,127,512,510
725 10,074,187,936,558,100
726 10,177,363,370,888,400
727 10,280,332,931,441,700
728 10,383,069,974,340,200
729 10,485,547,653,279,600
730 10,587,738,999,328,200
731 10,689,616,863,061,700
732 10,791,153,995,382,700
733 10,892,322,989,344,500
734 10,993,096,361,994,600
735 11,093,446,495,874,100
736 11,193,345,721,807,600
737 11,292,766,260,101,800
738 11,391,680,304,241,600
739 11,490,059,961,730,100
740 11,587,877,338,692,700
741 11,685,104,480,338,700
742 11,781,713,456,388,200
743 11,877,676,301,179,600
744 11,972,965,099,875,100
745 12,067,551,928,151,000
746 12,161,408,939,178,500
747 12,254,508,302,876,300
748 12,346,822,293,578,000
749 12,438,323,228,868,000
750 12,528,983,557,886,000
751 12,618,775,799,682,300
752 12,707,672,632,154,500
753 12,795,646,829,899,000
754 12,882,671,353,685,000
755 12,968,719,287,824,400
756 13,053,763,930,133,200
757 13,137,778,728,752,300
758 13,220,737,372,587,100
759 13,302,613,727,557,800
760 13,383,381,927,422,100
761 13,463,016,309,475,000
762 13,541,491,505,700,400
763 13,618,782,377,856,200
764 13,694,864,108,945,900
765 13,769,712,137,666,200
766 13,843,302,250,100,100
767 13,915,610,513,550,400
768 13,986,613,368,402,600
769 14,056,287,561,278,000
770 14,124,610,237,055,900
771 14,191,558,871,327,400
772 14,257,111,362,481,200
773 14,321,245,963,481,600
774 14,383,941,373,963,900
775 14,445,176,671,275,600
776 14,504,931,402,574,800
777 14,563,185,515,116,400
778 14,619,919,448,256,100
779 14,675,114,063,011,100
780 14,728,750,733,921,000
781 14,780,811,277,822,800
782 14,831,278,045,561,900
783 14,880,133,849,969,200
784 14,927,362,057,331,000
785 14,972,946,514,599,900
786 15,016,871,640,574,700
787 15,059,122,352,292,200
788 15,099,684,155,916,700
789 15,138,543,072,303,300
790 15,175,685,727,506,900
791 15,211,099,277,556,400
792 15,244,771,498,541,800
793 15,276,690,710,548,100
794 15,306,845,867,323,800
795 15,335,226,479,373,200
796 15,361,822,703,122,200
797 15,386,625,263,213,800
798 15,409,625,541,131,800
799 15,430,815,496,656,900
800 15,450,187,755,959,900


Source: https://sites.google.com/view/krapstuff/keno
"For with much wisdom comes much sorrow." -- Ecclesiastes 1:18 (NIV)
Wizard
Administrator
Wizard
  • Threads: 1493
  • Posts: 26485
Joined: Oct 14, 2009
May 20th, 2019 at 3:40:29 PM permalink
Here is the rest of the table.


801 15,467,735,532,221,000
802 15,483,452,713,112,300
803 15,497,333,780,632,900
804 15,509,373,897,951,000
805 15,519,568,828,416,200
806 15,527,915,021,776,200
807 15,534,409,532,379,100
808 15,539,050,104,692,300
809 15,541,835,090,752,100
810 15,542,763,534,960,500
811 15,541,835,090,752,100
812 15,539,050,104,692,300
813 15,534,409,532,379,100
814 15,527,915,021,776,200
815 15,519,568,828,416,200
816 15,509,373,897,951,000
817 15,497,333,780,632,900
818 15,483,452,713,112,300
819 15,467,735,532,221,000
820 15,450,187,755,959,900
821 15,430,815,496,656,900
822 15,409,625,541,131,800
823 15,386,625,263,213,800
824 15,361,822,703,122,200
825 15,335,226,479,373,200
826 15,306,845,867,323,800
827 15,276,690,710,548,100
828 15,244,771,498,541,800
829 15,211,099,277,556,400
830 15,175,685,727,506,900
831 15,138,543,072,303,300
832 15,099,684,155,916,700
833 15,059,122,352,292,200
834 15,016,871,640,574,700
835 14,972,946,514,599,900
836 14,927,362,057,331,000
837 14,880,133,849,969,200
838 14,831,278,045,561,900
839 14,780,811,277,822,800
840 14,728,750,733,921,000
841 14,675,114,063,011,100
842 14,619,919,448,256,100
843 14,563,185,515,116,400
844 14,504,931,402,574,800
845 14,445,176,671,275,600
846 14,383,941,373,963,900
847 14,321,245,963,481,600
848 14,257,111,362,481,200
849 14,191,558,871,327,400
850 14,124,610,237,055,900
851 14,056,287,561,278,000
852 13,986,613,368,402,600
853 13,915,610,513,550,400
854 13,843,302,250,100,100
855 13,769,712,137,666,200
856 13,694,864,108,945,900
857 13,618,782,377,856,200
858 13,541,491,505,700,400
859 13,463,016,309,475,000
860 13,383,381,927,422,100
861 13,302,613,727,557,800
862 13,220,737,372,587,100
863 13,137,778,728,752,300
864 13,053,763,930,133,200
865 12,968,719,287,824,400
866 12,882,671,353,685,000
867 12,795,646,829,899,000
868 12,707,672,632,154,500
869 12,618,775,799,682,300
870 12,528,983,557,886,000
871 12,438,323,228,868,000
872 12,346,822,293,578,000
873 12,254,508,302,876,300
874 12,161,408,939,178,500
875 12,067,551,928,151,000
876 11,972,965,099,875,100
877 11,877,676,301,179,600
878 11,781,713,456,388,200
879 11,685,104,480,338,700
880 11,587,877,338,692,700
881 11,490,059,961,730,100
882 11,391,680,304,241,600
883 11,292,766,260,101,800
884 11,193,345,721,807,600
885 11,093,446,495,874,100
886 10,993,096,361,994,600
887 10,892,322,989,344,500
888 10,791,153,995,382,700
889 10,689,616,863,061,700
890 10,587,738,999,328,200
891 10,485,547,653,279,600
892 10,383,069,974,340,200
893 10,280,332,931,441,700
894 10,177,363,370,888,400
895 10,074,187,936,558,100
896 9,970,833,127,512,510
897 9,867,325,219,249,980
898 9,763,690,321,033,400
899 9,659,954,298,267,300
900 9,556,142,829,555,640
901 9,452,281,330,191,890
902 9,348,395,008,996,000
903 9,244,508,792,945,670
904 9,140,647,383,762,520
905 9,036,835,183,748,460
906 8,933,096,352,127,630
907 8,829,454,732,072,480
908 8,725,933,906,847,840
909 8,622,557,128,046,050
910 8,519,347,371,498,560
911 8,416,327,266,778,080
912 8,313,519,152,881,930
913 8,210,945,008,978,190
914 8,108,626,509,899,880
915 8,006,584,958,150,180
916 7,904,841,339,170,870
917 7,803,416,254,655,220
918 7,702,329,977,589,080
919 7,601,602,386,843,980
920 7,501,253,022,027,260
921 7,401,301,019,361,720
922 7,301,765,166,303,070
923 7,202,663,838,749,380
924 7,104,015,055,421,620
925 7,005,836,416,368,480
926 6,908,145,157,139,450
927 6,810,958,088,586,950
928 6,714,291,650,789,160
929 6,618,161,854,183,300
930 6,522,584,333,229,140
931 6,427,574,288,835,360
932 6,333,146,541,790,360
933 6,239,315,476,476,150
934 6,146,095,094,019,120
935 6,053,498,957,320,960
936 5,961,540,243,918,360
937 5,870,231,692,286,640
938 5,779,585,654,429,600
939 5,689,614,043,446,160
940 5,600,328,385,803,350
941 5,511,739,770,187,560
942 5,423,858,899,445,150
943 5,336,696,040,671,980
944 5,250,261,076,840,900
945 5,164,563,458,113,970
946 5,079,612,253,106,590
947 4,995,416,101,440,400
948 4,911,983,264,628,410
949 4,829,321,579,817,140
950 4,747,438,510,307,980
951 4,666,341,100,470,600
952 4,586,036,025,851,500
953 4,506,529,549,270,560
954 4,427,827,570,497,980
955 4,349,935,583,482,260
956 4,272,858,725,610,060
957 4,196,601,736,043,210
958 4,121,169,004,511,070
959 4,046,564,530,766,010
960 3,972,791,972,891,010
961 3,899,854,607,818,590
962 3,827,755,379,165,350
963 3,756,496,858,791,470
964 3,686,081,294,113,980
965 3,616,510,570,712,250
966 3,547,786,259,101,070
967 3,479,909,578,328,190
968 3,412,881,442,219,840
969 3,346,702,423,944,310
970 3,281,372,801,682,290
971 3,216,892,524,160,980
972 3,153,261,255,732,850
973 3,090,478,342,825,040
974 3,028,542,858,437,460
975 2,967,453,569,482,800
976 2,907,208,980,659,850
977 2,847,807,302,685,920
978 2,789,246,495,528,880
979 2,731,524,237,478,950
980 2,674,637,967,752,940
981 2,618,584,856,379,090
982 2,563,361,846,130,740
983 2,508,965,623,226,930
984 2,455,392,658,582,210
985 2,402,639,179,270,460
986 2,350,701,209,103,640
987 2,299,574,540,834,780
988 2,249,254,776,027,770
989 2,199,737,298,000,370
990 2,151,017,310,973,090
991 2,103,089,813,703,010
992 2,055,949,637,927,470
993 2,009,591,422,664,540
994 1,964,009,651,915,660
995 1,919,198,629,635,730
996 1,875,152,516,686,540
997 1,831,865,306,428,570
998 1,789,330,860,941,280
999 1,747,542,887,215,360
1000 1,706,494,972,609,520
1001 1,666,180,561,645,290
1002 1,626,592,990,693,830
1003 1,587,725,465,329,520
1004 1,549,571,094,266,050
1005 1,512,122,867,248,400
1006 1,475,373,688,210,190
1007 1,439,316,353,708,700
1008 1,403,943,585,301,200
1009 1,369,248,008,480,890
1010 1,335,222,184,292,440
1011 1,301,858,588,751,630
1012 1,269,149,643,677,450
1013 1,237,087,696,599,980
1014 1,205,665,050,808,210
1015 1,174,873,945,708,900
1016 1,144,706,586,113,290
1017 1,115,155,123,030,640
1018 1,086,211,682,173,950
1019 1,057,868,345,195,040
1020 1,030,117,177,412,690
1021 1,002,950,209,453,770
1022 976,359,464,227,661
1023 950,336,938,960,611
1024 924,874,631,401,209
1025 899,964,522,255,079
1026 875,598,600,626,154
1027 851,768,846,820,577
1028 828,467,257,050,350
1029 805,685,826,594,439
1030 783,416,573,752,126
1031 761,651,523,370,470
1032 740,382,730,055,707
1033 719,602,262,037,791
1034 699,302,223,666,421
1035 679,474,739,604,064
1036 660,111,976,597,920
1037 641,206,128,010,489
1038 622,749,434,876,858
1039 604,734,170,747,184
1040 587,152,662,058,258
1041 569,997,273,279,789
1042 553,260,426,593,322
1043 536,934,587,353,256
1044 521,012,283,085,036
1045 505,486,089,237,058
1046 490,348,647,526,558
1047 475,592,651,976,889
1048 461,210,866,607,609
1049 447,196,111,767,398
1050 433,541,281,180,913
1051 420,239,328,556,562
1052 407,283,284,019,903
1053 394,666,240,991,089
1054 382,381,372,002,154
1055 370,421,915,855,964
1056 358,781,192,841,696
1057 347,452,592,155,562
1058 336,429,586,543,371
1059 325,705,719,979,621
1060 315,274,621,737,283
1061 305,129,994,336,046
1062 295,265,627,053,394
1063 285,675,384,124,940
1064 276,353,217,726,061
1065 267,293,156,421,037
1066 258,489,317,615,563
1067 249,935,896,266,269
1068 241,627,176,819,381
1069 233,557,522,163,930
1070 225,721,385,084,601
1071 218,113,297,456,569
1072 210,727,881,214,639
1073 203,559,837,800,120
1074 196,603,958,660,512
1075 189,855,114,934,011
1076 183,308,267,507,535
1077 176,958,456,936,318
1078 170,800,813,062,207
1079 164,830,545,178,892
1080 159,042,951,225,485
1081 153,433,408,182,971
1082 147,997,380,868,998
1083 142,730,412,563,985
1084 137,628,133,409,981
1085 132,686,251,275,763
1086 127,900,559,773,363
1087 123,266,929,349,329
1088 118,781,314,944,316
1089 114,439,747,308,027
1090 110,238,340,304,328
1091 106,173,282,459,017
1092 102,240,843,924,128
1093 98,437,368,245,632
1094 94,759,279,010,024
1095 91,203,071,830,294
1096 87,765,320,677,722
1097 84,442,670,093,739
1098 81,231,841,219,122
1099 78,129,624,219,915
1100 75,132,884,036,461
1101 72,238,553,020,681
1102 69,443,636,407,019
1103 66,745,205,168,528
1104 64,140,401,221,500
1105 61,626,430,488,492
1106 59,200,567,856,702
1107 56,860,150,445,667
1108 54,602,582,321,801
1109 52,425,327,976,532
1110 50,325,916,807,381
1111 48,301,936,795,528
1112 46,351,038,772,509
1113 44,470,930,294,083
1114 42,659,379,693,854
1115 40,914,210,159,126
1116 39,233,303,581,272
1117 37,614,594,822,327
1118 36,056,075,379,039
1119 34,555,787,837,266
1120 33,111,829,351,342
1121 31,722,346,290,416
1122 30,385,537,541,434
1123 29,099,649,337,087
1124 27,862,978,398,196
1125 26,673,866,939,713
1126 25,530,705,653,070
1127 24,431,928,894,154
1128 23,376,017,513,498
1129 22,361,494,215,565
1130 21,386,926,250,737
1131 20,450,920,942,555
1132 19,552,128,242,181
1133 18,689,236,426,307
1134 17,860,974,520,443
1135 17,066,108,157,849
1136 16,303,441,884,554
1137 15,571,815,175,325
1138 14,870,104,620,270
1139 14,197,220,100,303
1140 13,552,106,861,442
1141 12,933,741,839,899
1142 12,341,135,635,235
1143 11,773,328,981,483
1144 11,229,394,619,008
1145 10,708,433,913,262
1146 10,209,578,630,140
1147 9,731,987,693,363
1148 9,274,848,873,719
1149 8,837,375,680,898
1150 8,418,808,965,914
1151 8,018,413,948,848
1152 7,635,481,738,958
1153 7,269,326,489,361
1154 6,919,286,843,610
1155 6,584,723,213,704
1156 6,265,019,152,487
1157 5,959,578,755,669
1158 5,667,827,963,566
1159 5,389,212,078,927
1160 5,123,197,006,289
1161 4,869,266,881,582
1162 4,626,925,247,931
1163 4,395,692,797,816
1164 4,175,108,488,550
1165 3,964,727,389,027
1166 3,764,121,741,811
1167 3,572,878,910,940
1168 3,390,602,389,686
1169 3,216,909,849,508
1170 3,051,434,096,031
1171 2,893,821,211,946
1172 2,743,731,467,669
1173 2,600,837,554,495
1174 2,464,825,448,513
1175 2,335,392,734,382
1176 2,212,249,424,868
1177 2,095,116,368,246
1178 1,983,726,028,916
1179 1,877,820,975,168
1180 1,777,154,619,819
1181 1,681,489,788,194
1182 1,590,599,420,536
1183 1,504,265,214,046
1184 1,422,278,292,091
1185 1,344,437,917,067
1186 1,270,552,124,993
1187 1,200,436,509,276
1188 1,133,914,822,434
1189 1,070,817,824,773
1190 1,010,983,857,544
1191 954,257,753,733
1192 900,491,381,529
1193 849,542,617,012
1194 801,275,859,118
1195 755,561,059,108
1196 712,274,211,009
1197 671,296,435,063
1198 632,514,442,353
1199 595,819,672,189
1200 561,108,732,221
1201 528,282,584,833
1202 497,246,966,079
1203 467,911,618,783
1204 440,190,689,221
1205 414,002,006,749
1206 389,267,459,099
1207 365,912,314,318
1208 343,865,577,844
1209 323,059,354,425
1210 303,429,185,857
1211 284,913,452,879
1212 267,453,694,512
1213 250,994,045,973
1214 235,481,542,190
1215 220,865,589,927
1216 207,098,254,570
1217 194,133,766,301
1218 181,928,790,813
1219 170,441,966,142
1220 159,634,159,836
1221 149,468,034,620
1222 139,908,291,043
1223 130,921,262,045
1224 122,475,141,756
1225 114,539,605,867
1226 107,086,028,628
1227 100,087,127,594
1228 93,517,168,095
1229 87,351,632,249
1230 81,567,411,419
1231 76,142,496,873
1232 71,056,162,189
1233 66,288,674,187
1234 61,821,464,449
1235 57,636,860,670
1236 53,718,247,833
1237 50,049,817,550
1238 46,616,720,518
1239 43,404,832,757
1240 40,400,898,786
1241 37,592,314,751
1242 34,967,262,684
1243 32,514,508,546
1244 30,223,529,082
1245 28,084,323,773
1246 26,087,533,643
1247 24,224,267,278
1248 22,486,212,019
1249 20,865,472,184
1250 19,354,673,930
1251 17,946,814,901
1252 16,635,362,300
1253 15,414,114,032
1254 14,277,290,236
1255 13,219,404,469
1256 12,235,349,889
1257 11,320,279,729
1258 10,469,687,646
1259 9,679,297,643
1260 8,945,138,936
1261 8,263,443,950
1262 7,630,718,671
1263 7,043,648,186
1264 6,499,162,145
1265 5,994,347,965
1266 5,526,511,602
1267 5,093,097,335
1268 4,691,744,778
1269 4,320,214,692
1270 3,976,441,876
1271 3,658,467,191
1272 3,364,486,581
1273 3,092,788,314
1274 2,841,798,846
1275 2,610,024,936
1276 2,396,096,082
1277 2,198,711,618
1278 2,016,679,881
1279 1,848,869,496
1280 1,694,245,990
1281 1,551,826,889
1282 1,420,715,489
1283 1,300,059,954
1284 1,189,084,411
1285 1,087,051,347
1286 993,290,571
1287 907,164,667
1288 828,095,629
1289 755,533,468
1290 688,980,643
1291 627,963,266
1292 572,053,851
1293 520,844,871
1294 473,969,587
1295 431,078,138
1296 391,856,581
1297 356,005,000
1298 323,255,164
1299 293,350,487
1300 266,062,168
1301 241,171,101
1302 218,482,569
1303 197,809,729
1304 178,987,225
1305 161,856,093
1306 146,276,124
1307 132,112,328
1308 119,246,146
1309 107,563,103
1310 96,963,169
1311 87,349,501
1312 78,637,842
1313 70,746,444
1314 63,604,540
1315 57,143,201
1316 51,303,148
1317 46,026,431
1318 41,263,462
1319 36,965,620
1320 33,091,578
1321 29,600,587
1322 26,458,297
1323 23,630,664
1324 21,089,176
1325 18,805,464
1326 16,755,957
1327 14,917,024
1328 13,269,250
1329 11,793,028
1330 10,472,375
1331 9,291,060
1332 8,236,001
1333 7,293,767
1334 6,453,684
1335 5,704,686
1336 5,038,070
1337 4,444,748
1338 3,917,670
1339 3,449,359
1340 3,034,135
1341 2,665,885
1342 2,340,024
1343 2,051,569
1344 1,796,855
1345 1,571,812
1346 1,373,524
1347 1,198,689
1348 1,044,984
1349 909,741
1350 791,131
1351 686,983
1352 595,872
1353 516,054
1354 446,405
1355 385,528
1356 332,557
1357 286,364
1358 246,288
1359 211,428
1360 181,274
1361 155,112
1362 132,559
1363 113,039
1364 96,271
1365 81,801
1366 69,414
1367 58,754
1368 49,668
1369 41,869
1370 35,251
1371 29,588
1372 24,803
1373 20,722
1374 17,293
1375 14,375
1376 11,937
1377 9,871
1378 8,154
1379 6,703
1380 5,507
1381 4,498
1382 3,673
1383 2,980
1384 2,417
1385 1,946
1386 1,568
1387 1,251
1388 1,000
1389 791
1390 627
1391 490
1392 385
1393 297
1394 231
1395 176
1396 135
1397 101
1398 77
1399 56
1400 42
1401 30
1402 22
1403 15
1404 11
1405 7
1406 5
1407 3
1408 2
1409 1
1410 1
"For with much wisdom comes much sorrow." -- Ecclesiastes 1:18 (NIV)
7craps
7craps
  • Threads: 18
  • Posts: 1977
Joined: Jan 23, 2010
May 20th, 2019 at 4:18:20 PM permalink
Quote: Wizard

Interesting. I wish Sally were still around to convert that to more plain simple English. Can anyone else help?

Sally seems to be long gone (moved to Kentucky) but you can try to email her. She stopped responding to mine.

here is where the formula was found
https://math.stackexchange.com/questions/2167789/ways-of-getting-a-sum-of-s-with-m-unique-numbers-in-the-range-1-n

This subject (Keno sums) does not interest me at the time. Maybe I will look closer into the generating function and see how it can be easily used.
I do not have Mathematica (but can use pari/gp). arbitrary precision is nice when it works within reason.
enjoy
winsome johnny (not Win some johnny)
Wizard
Administrator
Wizard
  • Threads: 1493
  • Posts: 26485
Joined: Oct 14, 2009
May 20th, 2019 at 4:21:47 PM permalink
I'm afraid I just don't get that formula. I do have Sally's address, but don't want to bother her over this.
"For with much wisdom comes much sorrow." -- Ecclesiastes 1:18 (NIV)
7craps
7craps
  • Threads: 18
  • Posts: 1977
Joined: Jan 23, 2010
May 20th, 2019 at 4:28:27 PM permalink
Quote: Wizard

I'm afraid I just don't get that formula.

prod(i=1, 80, 1 + x*y^i);
is a product function (like the sum())
i starts at 1 and goes to 80 and 1 + x*y^i is just the expression
that link I gave seems to give a better description than I could.
I am 100% certain you can code this in c++

back to baseball and NBA tonite

I do remember verifying her data at the time (I just did not do it myself)
winsome johnny (not Win some johnny)
Ayecarumba
Ayecarumba
  • Threads: 236
  • Posts: 6763
Joined: Nov 17, 2009
May 20th, 2019 at 5:00:27 PM permalink
Musings of a completely untrained mind...

Since there's only one way each to make the extreme totals 210 (balls 1-20) and 1410 (balls 61-80), I was expecting to see a perfectly smooth distribution of the instances of combinations to make the other totals with a peak at ~810. However, there are some totals with many more ways to make them compared to the one-off totals next to them. Is it because they are Primes? For example, there are 578 more ways to make a total of 456 than 455, but only 80 more ways to make 457 than 456. Can the existence of rarer totals be useful?

My gut tells me that a quick solution will involve assigning a weight to each ball reflecting the number of totals, not necessarily the totals themselves, that they can be part of. For example, ball #1 will be more valuable than ball #2 since it can be used to make the smallest incremental even and odd totals, but #2 can only make odd totals the next odd total, or even totals the next even.

Of course, that could be the Del Taco $4 value box I had for lunch talking.
Simplicity is the ultimate sophistication - Leonardo da Vinci
ThatDonGuy
ThatDonGuy
  • Threads: 117
  • Posts: 6267
Joined: Jun 22, 2011
May 20th, 2019 at 5:00:51 PM permalink
Quote: Wizard

I'm afraid I just don't get that formula. I do have Sally's address, but don't want to bother her over this.


I think I sort of do.
The formula is the product of:
1 + x y
1 + x y2
1 + x y3
1 + x y4
...
1 + x y79
1 + x y80

If you draw 20 particular balls, multiply the product of the left sides of the terms of the balls you did not draw and the right sides of the balls you did draw together; you will get x20 and ysum of the numbers.
To determine how many times 20 balls add up to some number N, count the number of times x20 yN is such a product.

How to get such a count without having to go through all 280 products, I don't know, and presumably, that's why things like Mathematica exist.
Wizard
Administrator
Wizard
  • Threads: 1493
  • Posts: 26485
Joined: Oct 14, 2009
May 20th, 2019 at 5:02:54 PM permalink
Quote: 7craps

prod(i=1, 80, 1 + x*y^i);
is a product function (like the sum())
i starts at 1 and goes to 80 and 1 + x*y^i is just the expression
that link I gave seems to give a better description than I could.
I am 100% certain you can code this in c++



I'm afraid I just don't understand that post. Hopefully this thread will get more interest from the other math geniuses of the forum.
"For with much wisdom comes much sorrow." -- Ecclesiastes 1:18 (NIV)
7craps
7craps
  • Threads: 18
  • Posts: 1977
Joined: Jan 23, 2010
May 20th, 2019 at 5:25:31 PM permalink
Quote: ThatDonGuy

How to get such a count without having to go through all 280 products, I don't know, and presumably, that's why things like Mathematica exist.

what you say is so true.
I looked for the original pari/gp text file that is created from that formula and I do not have it and do not want to make it. I think it was very large and a program as notepad++ only could open it and make things happen.
added: I made that file and it is only 1.9MB in a text file and took less than 1 second to make.

Here is what I do have for x^20 (draw 20)
y^sum is the sum and the coefficient is the number of ways that sum can occur.
this can be compared to by one who really wants to do this

This concept is about partitions

here is 2 numbers and 3 numbers expanded and it should make sense.
gp > a=prod(i=1, 2, 1 + x*y^i);
gp > b=prod(i=1, 3, 1 + x*y^i);
gp > a
%3 = y^3*x^2 + (y^2 + y)*x + 1
gp > b
%4 = y^6*x^3 + (y^5 + y^4 + y^3)*x^2 + (y^3 + y^2 + y)*x + 1

a=prod(i=1, 2, 1 + x*y^i); means we only have 2 numbers (1 and 2)
y^3*x^2 + (y^2 + y)*x + 1 means drawing 2 numbers (x^2) we have only 1 sum and that is y^3 = 3 and only 1 way to get it (1+2)
(y^2 + y)*x means we draw 1 number (x) and the sums are 2 and 1 and 1 way to get each

same concept with numbers {1,2,3}
x^20
1*y^1410
1*y^1409
2*y^1408
3*y^1407
5*y^1406
7*y^1405
11*y^1404
15*y^1403
22*y^1402
30*y^1401
42*y^1400
56*y^1399
77*y^1398
101*y^1397
135*y^1396
176*y^1395
231*y^1394
297*y^1393
385*y^1392
490*y^1391
627*y^1390
791*y^1389
1000*y^1388
1251*y^1387
1568*y^1386
1946*y^1385
2417*y^1384
2980*y^1383
3673*y^1382
4498*y^1381
5507*y^1380
6703*y^1379
8154*y^1378
9871*y^1377
11937*y^1376
14375*y^1375
17293*y^1374
20722*y^1373
24803*y^1372
29588*y^1371
35251*y^1370
41869*y^1369
49668*y^1368
58754*y^1367
69414*y^1366
81801*y^1365
96271*y^1364
113039*y^1363
132559*y^1362
155112*y^1361
181274*y^1360
211428*y^1359
246288*y^1358
286364*y^1357
332557*y^1356
385528*y^1355
446405*y^1354
516054*y^1353
595872*y^1352
686983*y^1351
791131*y^1350
909741*y^1349
1044984*y^1348
1198689*y^1347
1373524*y^1346
1571812*y^1345
1796855*y^1344
2051569*y^1343
2340024*y^1342
2665885*y^1341
3034135*y^1340
3449359*y^1339
3917670*y^1338
4444748*y^1337
5038070*y^1336
5704686*y^1335
6453684*y^1334
7293767*y^1333
8236001*y^1332
9291060*y^1331
10472375*y^1330
11793028*y^1329
13269250*y^1328
14917024*y^1327
16755957*y^1326
18805464*y^1325
21089176*y^1324
23630664*y^1323
26458297*y^1322
29600587*y^1321
33091578*y^1320
36965620*y^1319
41263462*y^1318
46026431*y^1317
51303148*y^1316
57143201*y^1315
63604540*y^1314
70746444*y^1313
78637842*y^1312
87349501*y^1311
96963169*y^1310
107563103*y^1309
119246146*y^1308
132112328*y^1307
146276124*y^1306
161856093*y^1305
178987225*y^1304
197809729*y^1303
218482569*y^1302
241171101*y^1301
266062168*y^1300
293350487*y^1299
323255164*y^1298
356005000*y^1297
391856581*y^1296
431078138*y^1295
473969587*y^1294
520844871*y^1293
572053851*y^1292
627963266*y^1291
688980643*y^1290
755533468*y^1289
828095629*y^1288
907164667*y^1287
993290571*y^1286
1087051347*y^1285
1189084411*y^1284
1300059954*y^1283
1420715489*y^1282
1551826889*y^1281
1694245990*y^1280
1848869496*y^1279
2016679881*y^1278
2198711618*y^1277
2396096082*y^1276
2610024936*y^1275
2841798846*y^1274
3092788314*y^1273
3364486581*y^1272
3658467191*y^1271
3976441876*y^1270
4320214692*y^1269
4691744778*y^1268
5093097335*y^1267
5526511602*y^1266
5994347965*y^1265
6499162145*y^1264
7043648186*y^1263
7630718671*y^1262
8263443950*y^1261
8945138936*y^1260
9679297643*y^1259
10469687646*y^1258
11320279729*y^1257
12235349889*y^1256
13219404469*y^1255
14277290236*y^1254
15414114032*y^1253
16635362300*y^1252
17946814901*y^1251
19354673930*y^1250
20865472184*y^1249
22486212019*y^1248
24224267278*y^1247
26087533643*y^1246
28084323773*y^1245
30223529082*y^1244
32514508546*y^1243
34967262684*y^1242
37592314751*y^1241
40400898786*y^1240
43404832757*y^1239
46616720518*y^1238
50049817550*y^1237
53718247833*y^1236
57636860670*y^1235
61821464449*y^1234
66288674187*y^1233
71056162189*y^1232
76142496873*y^1231
81567411419*y^1230
87351632249*y^1229
93517168095*y^1228
100087127594*y^1227
107086028628*y^1226
114539605867*y^1225
122475141756*y^1224
130921262045*y^1223
139908291043*y^1222
149468034620*y^1221
159634159836*y^1220
170441966142*y^1219
181928790813*y^1218
194133766301*y^1217
207098254570*y^1216
220865589927*y^1215
235481542190*y^1214
250994045973*y^1213
267453694512*y^1212
284913452879*y^1211
303429185857*y^1210
323059354425*y^1209
343865577844*y^1208
365912314318*y^1207
389267459099*y^1206
414002006749*y^1205
440190689221*y^1204
467911618783*y^1203
497246966079*y^1202
528282584833*y^1201
561108732221*y^1200
595819672189*y^1199
632514442353*y^1198
671296435063*y^1197
712274211009*y^1196
755561059108*y^1195
801275859118*y^1194
849542617012*y^1193
900491381529*y^1192
954257753733*y^1191
1010983857544*y^1190
1070817824773*y^1189
1133914822434*y^1188
1200436509276*y^1187
1270552124993*y^1186
1344437917067*y^1185
1422278292091*y^1184
1504265214046*y^1183
1590599420536*y^1182
1681489788194*y^1181
1777154619819*y^1180
1877820975168*y^1179
1983726028916*y^1178
2095116368246*y^1177
2212249424868*y^1176
2335392734382*y^1175
2464825448513*y^1174
2600837554495*y^1173
2743731467669*y^1172
2893821211946*y^1171
3051434096031*y^1170
3216909849508*y^1169
3390602389686*y^1168
3572878910940*y^1167
3764121741811*y^1166
3964727389027*y^1165
4175108488550*y^1164
4395692797816*y^1163
4626925247931*y^1162
4869266881582*y^1161
5123197006289*y^1160
5389212078927*y^1159
5667827963566*y^1158
5959578755669*y^1157
6265019152487*y^1156
6584723213704*y^1155
6919286843610*y^1154
7269326489361*y^1153
7635481738958*y^1152
8018413948848*y^1151
8418808965914*y^1150
8837375680898*y^1149
9274848873719*y^1148
9731987693363*y^1147
10209578630140*y^1146
10708433913262*y^1145
11229394619008*y^1144
11773328981483*y^1143
12341135635235*y^1142
12933741839899*y^1141
13552106861442*y^1140
14197220100303*y^1139
14870104620270*y^1138
15571815175325*y^1137
16303441884554*y^1136
17066108157849*y^1135
17860974520443*y^1134
18689236426307*y^1133
19552128242181*y^1132
20450920942555*y^1131
21386926250737*y^1130
22361494215565*y^1129
23376017513498*y^1128
24431928894154*y^1127
25530705653070*y^1126
26673866939713*y^1125
27862978398196*y^1124
29099649337087*y^1123
30385537541434*y^1122
31722346290416*y^1121
33111829351342*y^1120
34555787837266*y^1119
36056075379039*y^1118
37614594822327*y^1117
39233303581272*y^1116
40914210159126*y^1115
42659379693854*y^1114
44470930294083*y^1113
46351038772509*y^1112
48301936795528*y^1111
50325916807381*y^1110
52425327976532*y^1109
54602582321801*y^1108
56860150445667*y^1107
59200567856702*y^1106
61626430488492*y^1105
64140401221500*y^1104
66745205168528*y^1103
69443636407019*y^1102
72238553020681*y^1101
75132884036461*y^1100
78129624219915*y^1099
81231841219122*y^1098
84442670093739*y^1097
87765320677722*y^1096
91203071830294*y^1095
94759279010024*y^1094
98437368245632*y^1093
102240843924128*y^1092
106173282459017*y^1091
110238340304328*y^1090
114439747308027*y^1089
118781314944316*y^1088
123266929349329*y^1087
127900559773363*y^1086
132686251275763*y^1085
137628133409981*y^1084
142730412563985*y^1083
147997380868998*y^1082
153433408182971*y^1081
159042951225485*y^1080
164830545178892*y^1079
170800813062207*y^1078
176958456936318*y^1077
183308267507535*y^1076
189855114934011*y^1075
196603958660512*y^1074
203559837800120*y^1073
210727881214639*y^1072
218113297456569*y^1071
225721385084601*y^1070
233557522163930*y^1069
241627176819381*y^1068
249935896266269*y^1067
258489317615563*y^1066
267293156421037*y^1065
276353217726061*y^1064
285675384124940*y^1063
295265627053394*y^1062
305129994336046*y^1061
315274621737283*y^1060
325705719979621*y^1059
336429586543371*y^1058
347452592155562*y^1057
358781192841696*y^1056
370421915855964*y^1055
382381372002154*y^1054
394666240991089*y^1053
407283284019903*y^1052
420239328556562*y^1051
433541281180913*y^1050
447196111767398*y^1049
461210866607609*y^1048
475592651976889*y^1047
490348647526558*y^1046
505486089237058*y^1045
521012283085036*y^1044
536934587353256*y^1043
553260426593322*y^1042
569997273279789*y^1041
587152662058258*y^1040
604734170747184*y^1039
622749434876858*y^1038
641206128010489*y^1037
660111976597920*y^1036
679474739604064*y^1035
699302223666421*y^1034
719602262037791*y^1033
740382730055707*y^1032
761651523370470*y^1031
783416573752126*y^1030
805685826594439*y^1029
828467257050350*y^1028
851768846820577*y^1027
875598600626154*y^1026
899964522255079*y^1025
924874631401209*y^1024
950336938960611*y^1023
976359464227661*y^1022
1002950209453776*y^1021
1030117177412699*y^1020
1057868345195045*y^1019
1086211682173956*y^1018
1115155123030642*y^1017
1144706586113291*y^1016
1174873945708909*y^1015
1205665050808215*y^1014
1237087696599984*y^1013
1269149643677458*y^1012
1301858588751637*y^1011
1335222184292444*y^1010
1369248008480895*y^1009
1403943585301204*y^1008
1439316353708705*y^1007
1475373688210195*y^1006
1512122867248405*y^1005
1549571094266055*y^1004
1587725465329523*y^1003
1626592990693830*y^1002
1666180561645293*y^1001
1706494972609528*y^1000
1747542887215360*y^999
1789330860941288*y^998
1831865306428571*y^997
1875152516686548*y^996
1919198629635734*y^995
1964009651915660*y^994
2009591422664545*y^993
2055949637927477*y^992
2103089813703016*y^991
2151017310973098*y^990
2199737298000372*y^989
2249254776027773*y^988
2299574540834782*y^987
2350701209103646*y^986
2402639179270463*y^985
2455392658582214*y^984
2508965623226933*y^983
2563361846130743*y^982
2618584856379097*y^981
2674637967752948*y^980
2731524237478957*y^979
2789246495528881*y^978
2847807302685929*y^977
2907208980659856*y^976
2967453569482805*y^975
3028542858437463*y^974
3090478342825046*y^973
3153261255732853*y^972
3216892524160984*y^971
3281372801682291*y^970
3346702423944313*y^969
3412881442219846*y^968
3479909578328192*y^967
3547786259101073*y^966
3616510570712255*y^965
3686081294113980*y^964
3756496858791478*y^963
3827755379165355*y^962
3899854607818599*y^961
3972791972891010*y^960
4046564530766019*y^959
4121169004511073*y^958
4196601736043217*y^957
4272858725610066*y^956
4349935583482269*y^955
4427827570497986*y^954
4506529549270567*y^953
4586036025851501*y^952
4666341100470600*y^951
4747438510307985*y^950
4829321579817147*y^949
4911983264628415*y^948
4995416101440403*y^947
5079612253106599*y^946
5164563458113973*y^945
5250261076840903*y^944
5336696040671980*y^943
5423858899445150*y^942
5511739770187560*y^941
5600328385803359*y^940
5689614043446161*y^939
5779585654429600*y^938
5870231692286645*y^937
5961540243918360*y^936
6053498957320960*y^935
6146095094019128*y^934
6239315476476153*y^933
6333146541790366*y^932
6427574288835366*y^931
6522584333229145*y^930
6618161854183309*y^929
6714291650789160*y^928
6810958088586958*y^927
6908145157139459*y^926
7005836416368480*y^925
7104015055421622*y^924
7202663838749386*y^923
7301765166303075*y^922
7401301019361722*y^921
7501253022027266*y^920
7601602386843987*y^919
7702329977589080*y^918
7803416254655222*y^917
7904841339170873*y^916
8006584958150181*y^915
8108626509899889*y^914
8210945008978195*y^913
8313519152881939*y^912
8416327266778085*y^911
8519347371498567*y^910
8622557128046053*y^909
8725933906847840*y^908
8829454732072484*y^907
8933096352127635*y^906
9036835183748460*y^905
9140647383762528*y^904
9244508792945678*y^903
9348395008996007*y^902
9452281330191897*y^901
9556142829555643*y^900
9659954298267308*y^899
9763690321033408*y^898
9867325219249981*y^897
9970833127512512*y^896
10074187936558122*y^895
10177363370888455*y^894
10280332931441733*y^893
10383069974340297*y^892
10485547653279647*y^891
10587738999328217*y^890
10689616863061737*y^889
10791153995382729*y^888
10892322989344543*y^887
10993096361994634*y^886
11093446495874170*y^885
11193345721807668*y^884
11292766260101878*y^883
11391680304241666*y^882
11490059961730183*y^881
11587877338692722*y^880
11685104480338701*y^879
11781713456388257*y^878
11877676301179640*y^877
11972965099875122*y^876
12067551928151070*y^875
12161408939178544*y^874
12254508302876326*y^873
12346822293578054*y^872
12438323228868060*y^871
12528983557886059*y^870
12618775799682379*y^869
12707672632154594*y^868
12795646829899064*y^867
12882671353685064*y^866
12968719287824486*y^865
13053763930133222*y^864
13137778728752338*y^863
13220737372587114*y^862
13302613727557896*y^861
13383381927422176*y^860
13463016309475063*y^859
13541491505700427*y^858
13618782377856223*y^857
13694864108945970*y^856
13769712137666260*y^855
13843302250100143*y^854
13915610513550433*y^853
13986613368402674*y^852
14056287561278035*y^851
14124610237055954*y^850
14191558871327486*y^849
14257111362481292*y^848
14321245963481667*y^847
14383941373963924*y^846
14445176671275633*y^845
14504931402574878*y^844
14563185515116493*y^843
14619919448256126*y^842
14675114063011110*y^841
14728750733921081*y^840
14780811277822859*y^839
14831278045561927*y^838
14880133849969280*y^837
14927362057331041*y^836
14972946514599961*y^835
15016871640574771*y^834
15059122352292240*y^833
15099684155916740*y^832
15138543072303315*y^831
15175685727506919*y^830
15211099277556439*y^829
15244771498541818*y^828
15276690710548182*y^827
15306845867323822*y^826
15335226479373283*y^825
15361822703122288*y^824
15386625263213894*y^823
15409625541131887*y^822
15430815496656959*y^821
15450187755959996*y^820
15467735532221058*y^819
15483452713112335*y^818
15497333780632956*y^817
15509373897951036*y^816
15519568828416239*y^815
15527915021776296*y^814
15534409532379142*y^813
15539050104692302*y^812
15541835090752144*y^811
15542763534960598*y^810
15541835090752144*y^809
15539050104692302*y^808
15534409532379142*y^807
15527915021776296*y^806
15519568828416239*y^805
15509373897951036*y^804
15497333780632956*y^803
15483452713112335*y^802
15467735532221058*y^801
15450187755959996*y^800
15430815496656959*y^799
15409625541131887*y^798
15386625263213894*y^797
15361822703122288*y^796
15335226479373283*y^795
15306845867323822*y^794
15276690710548182*y^793
15244771498541818*y^792
15211099277556439*y^791
15175685727506919*y^790
15138543072303315*y^789
15099684155916740*y^788
15059122352292240*y^787
15016871640574771*y^786
14972946514599961*y^785
14927362057331041*y^784
14880133849969280*y^783
14831278045561927*y^782
14780811277822859*y^781
14728750733921081*y^780
14675114063011110*y^779
14619919448256126*y^778
14563185515116493*y^777
14504931402574878*y^776
14445176671275633*y^775
14383941373963924*y^774
14321245963481667*y^773
14257111362481292*y^772
14191558871327486*y^771
14124610237055954*y^770
14056287561278035*y^769
13986613368402674*y^768
13915610513550433*y^767
13843302250100143*y^766
13769712137666260*y^765
13694864108945970*y^764
13618782377856223*y^763
13541491505700427*y^762
13463016309475063*y^761
13383381927422176*y^760
13302613727557896*y^759
13220737372587114*y^758
13137778728752338*y^757
13053763930133222*y^756
12968719287824486*y^755
12882671353685064*y^754
12795646829899064*y^753
12707672632154594*y^752
12618775799682379*y^751
12528983557886059*y^750
12438323228868060*y^749
12346822293578054*y^748
12254508302876326*y^747
12161408939178544*y^746
12067551928151070*y^745
11972965099875122*y^744
11877676301179640*y^743
11781713456388257*y^742
11685104480338701*y^741
11587877338692722*y^740
11490059961730183*y^739
11391680304241666*y^738
11292766260101878*y^737
11193345721807668*y^736
11093446495874170*y^735
10993096361994634*y^734
10892322989344543*y^733
10791153995382729*y^732
10689616863061737*y^731
10587738999328217*y^730
10485547653279647*y^729
10383069974340297*y^728
10280332931441733*y^727
10177363370888455*y^726
10074187936558122*y^725
9970833127512512*y^724
9867325219249981*y^723
9763690321033408*y^722
9659954298267308*y^721
9556142829555643*y^720
9452281330191897*y^719
9348395008996007*y^718
9244508792945678*y^717
9140647383762528*y^716
9036835183748460*y^715
8933096352127635*y^714
8829454732072484*y^713
8725933906847840*y^712
8622557128046053*y^711
8519347371498567*y^710
8416327266778085*y^709
8313519152881939*y^708
8210945008978195*y^707
8108626509899889*y^706
8006584958150181*y^705
7904841339170873*y^704
7803416254655222*y^703
7702329977589080*y^702
7601602386843987*y^701
7501253022027266*y^700
7401301019361722*y^699
7301765166303075*y^698
7202663838749386*y^697
7104015055421622*y^696
7005836416368480*y^695
6908145157139459*y^694
6810958088586958*y^693
6714291650789160*y^692
6618161854183309*y^691
6522584333229145*y^690
6427574288835366*y^689
6333146541790366*y^688
6239315476476153*y^687
6146095094019128*y^686
6053498957320960*y^685
5961540243918360*y^684
5870231692286645*y^683
5779585654429600*y^682
5689614043446161*y^681
5600328385803359*y^680
5511739770187560*y^679
5423858899445150*y^678
5336696040671980*y^677
5250261076840903*y^676
5164563458113973*y^675
5079612253106599*y^674
4995416101440403*y^673
4911983264628415*y^672
4829321579817147*y^671
4747438510307985*y^670
4666341100470600*y^669
4586036025851501*y^668
4506529549270567*y^667
4427827570497986*y^666
4349935583482269*y^665
4272858725610066*y^664
4196601736043217*y^663
4121169004511073*y^662
4046564530766019*y^661
3972791972891010*y^660
3899854607818599*y^659
3827755379165355*y^658
3756496858791478*y^657
3686081294113980*y^656
3616510570712255*y^655
3547786259101073*y^654
3479909578328192*y^653
3412881442219846*y^652
3346702423944313*y^651
3281372801682291*y^650
3216892524160984*y^649
3153261255732853*y^648
3090478342825046*y^647
3028542858437463*y^646
2967453569482805*y^645
2907208980659856*y^644
2847807302685929*y^643
2789246495528881*y^642
2731524237478957*y^641
2674637967752948*y^640
2618584856379097*y^639
2563361846130743*y^638
2508965623226933*y^637
2455392658582214*y^636
2402639179270463*y^635
2350701209103646*y^634
2299574540834782*y^633
2249254776027773*y^632
2199737298000372*y^631
2151017310973098*y^630
2103089813703016*y^629
2055949637927477*y^628
2009591422664545*y^627
1964009651915660*y^626
1919198629635734*y^625
1875152516686548*y^624
1831865306428571*y^623
1789330860941288*y^622
1747542887215360*y^621
1706494972609528*y^620
1666180561645293*y^619
1626592990693830*y^618
1587725465329523*y^617
1549571094266055*y^616
1512122867248405*y^615
1475373688210195*y^614
1439316353708705*y^613
1403943585301204*y^612
1369248008480895*y^611
1335222184292444*y^610
1301858588751637*y^609
1269149643677458*y^608
1237087696599984*y^607
1205665050808215*y^606
1174873945708909*y^605
1144706586113291*y^604
1115155123030642*y^603
1086211682173956*y^602
1057868345195045*y^601
1030117177412699*y^600
1002950209453776*y^599
976359464227661*y^598
950336938960611*y^597
924874631401209*y^596
899964522255079*y^595
875598600626154*y^594
851768846820577*y^593
828467257050350*y^592
805685826594439*y^591
783416573752126*y^590
761651523370470*y^589
740382730055707*y^588
719602262037791*y^587
699302223666421*y^586
679474739604064*y^585
660111976597920*y^584
641206128010489*y^583
622749434876858*y^582
604734170747184*y^581
587152662058258*y^580
569997273279789*y^579
553260426593322*y^578
536934587353256*y^577
521012283085036*y^576
505486089237058*y^575
490348647526558*y^574
475592651976889*y^573
461210866607609*y^572
447196111767398*y^571
433541281180913*y^570
420239328556562*y^569
407283284019903*y^568
394666240991089*y^567
382381372002154*y^566
370421915855964*y^565
358781192841696*y^564
347452592155562*y^563
336429586543371*y^562
325705719979621*y^561
315274621737283*y^560
305129994336046*y^559
295265627053394*y^558
285675384124940*y^557
276353217726061*y^556
267293156421037*y^555
258489317615563*y^554
249935896266269*y^553
241627176819381*y^552
233557522163930*y^551
225721385084601*y^550
218113297456569*y^549
210727881214639*y^548
203559837800120*y^547
196603958660512*y^546
189855114934011*y^545
183308267507535*y^544
176958456936318*y^543
170800813062207*y^542
164830545178892*y^541
159042951225485*y^540
153433408182971*y^539
147997380868998*y^538
142730412563985*y^537
137628133409981*y^536
132686251275763*y^535
127900559773363*y^534
123266929349329*y^533
118781314944316*y^532
114439747308027*y^531
110238340304328*y^530
106173282459017*y^529
102240843924128*y^528
98437368245632*y^527
94759279010024*y^526
91203071830294*y^525
87765320677722*y^524
84442670093739*y^523
81231841219122*y^522
78129624219915*y^521
75132884036461*y^520
72238553020681*y^519
69443636407019*y^518
66745205168528*y^517
64140401221500*y^516
61626430488492*y^515
59200567856702*y^514
56860150445667*y^513
54602582321801*y^512
52425327976532*y^511
50325916807381*y^510
48301936795528*y^509
46351038772509*y^508
44470930294083*y^507
42659379693854*y^506
40914210159126*y^505
39233303581272*y^504
37614594822327*y^503
36056075379039*y^502
34555787837266*y^501
33111829351342*y^500
31722346290416*y^499
30385537541434*y^498
29099649337087*y^497
27862978398196*y^496
26673866939713*y^495
25530705653070*y^494
24431928894154*y^493
23376017513498*y^492
22361494215565*y^491
21386926250737*y^490
20450920942555*y^489
19552128242181*y^488
18689236426307*y^487
17860974520443*y^486
17066108157849*y^485
16303441884554*y^484
15571815175325*y^483
14870104620270*y^482
14197220100303*y^481
13552106861442*y^480
12933741839899*y^479
12341135635235*y^478
11773328981483*y^477
11229394619008*y^476
10708433913262*y^475
10209578630140*y^474
9731987693363*y^473
9274848873719*y^472
8837375680898*y^471
8418808965914*y^470
8018413948848*y^469
7635481738958*y^468
7269326489361*y^467
6919286843610*y^466
6584723213704*y^465
6265019152487*y^464
5959578755669*y^463
5667827963566*y^462
5389212078927*y^461
5123197006289*y^460
4869266881582*y^459
4626925247931*y^458
4395692797816*y^457
4175108488550*y^456
3964727389027*y^455
3764121741811*y^454
3572878910940*y^453
3390602389686*y^452
3216909849508*y^451
3051434096031*y^450
2893821211946*y^449
2743731467669*y^448
2600837554495*y^447
2464825448513*y^446
2335392734382*y^445
2212249424868*y^444
2095116368246*y^443
1983726028916*y^442
1877820975168*y^441
1777154619819*y^440
1681489788194*y^439
1590599420536*y^438
1504265214046*y^437
1422278292091*y^436
1344437917067*y^435
1270552124993*y^434
1200436509276*y^433
1133914822434*y^432
1070817824773*y^431
1010983857544*y^430
954257753733*y^429
900491381529*y^428
849542617012*y^427
801275859118*y^426
755561059108*y^425
712274211009*y^424
671296435063*y^423
632514442353*y^422
595819672189*y^421
561108732221*y^420
528282584833*y^419
497246966079*y^418
467911618783*y^417
440190689221*y^416
414002006749*y^415
389267459099*y^414
365912314318*y^413
343865577844*y^412
323059354425*y^411
303429185857*y^410
284913452879*y^409
267453694512*y^408
250994045973*y^407
235481542190*y^406
220865589927*y^405
207098254570*y^404
194133766301*y^403
181928790813*y^402
170441966142*y^401
159634159836*y^400
149468034620*y^399
139908291043*y^398
130921262045*y^397
122475141756*y^396
114539605867*y^395
107086028628*y^394
100087127594*y^393
93517168095*y^392
87351632249*y^391
81567411419*y^390
76142496873*y^389
71056162189*y^388
66288674187*y^387
61821464449*y^386
57636860670*y^385
53718247833*y^384
50049817550*y^383
46616720518*y^382
43404832757*y^381
40400898786*y^380
37592314751*y^379
34967262684*y^378
32514508546*y^377
30223529082*y^376
28084323773*y^375
26087533643*y^374
24224267278*y^373
22486212019*y^372
20865472184*y^371
19354673930*y^370
17946814901*y^369
16635362300*y^368
15414114032*y^367
14277290236*y^366
13219404469*y^365
12235349889*y^364
11320279729*y^363
10469687646*y^362
9679297643*y^361
8945138936*y^360
8263443950*y^359
7630718671*y^358
7043648186*y^357
6499162145*y^356
5994347965*y^355
5526511602*y^354
5093097335*y^353
4691744778*y^352
4320214692*y^351
3976441876*y^350
3658467191*y^349
3364486581*y^348
3092788314*y^347
2841798846*y^346
2610024936*y^345
2396096082*y^344
2198711618*y^343
2016679881*y^342
1848869496*y^341
1694245990*y^340
1551826889*y^339
1420715489*y^338
1300059954*y^337
1189084411*y^336
1087051347*y^335
993290571*y^334
907164667*y^333
828095629*y^332
755533468*y^331
688980643*y^330
627963266*y^329
572053851*y^328
520844871*y^327
473969587*y^326
431078138*y^325
391856581*y^324
356005000*y^323
323255164*y^322
293350487*y^321
266062168*y^320
241171101*y^319
218482569*y^318
197809729*y^317
178987225*y^316
161856093*y^315
146276124*y^314
132112328*y^313
119246146*y^312
107563103*y^311
96963169*y^310
87349501*y^309
78637842*y^308
70746444*y^307
63604540*y^306
57143201*y^305
51303148*y^304
46026431*y^303
41263462*y^302
36965620*y^301
33091578*y^300
29600587*y^299
26458297*y^298
23630664*y^297
21089176*y^296
18805464*y^295
16755957*y^294
14917024*y^293
13269250*y^292
11793028*y^291
10472375*y^290
9291060*y^289
8236001*y^288
7293767*y^287
6453684*y^286
5704686*y^285
5038070*y^284
4444748*y^283
3917670*y^282
3449359*y^281
3034135*y^280
2665885*y^279
2340024*y^278
2051569*y^277
1796855*y^276
1571812*y^275
1373524*y^274
1198689*y^273
1044984*y^272
909741*y^271
791131*y^270
686983*y^269
595872*y^268
516054*y^267
446405*y^266
385528*y^265
332557*y^264
286364*y^263
246288*y^262
211428*y^261
181274*y^260
155112*y^259
132559*y^258
113039*y^257
96271*y^256
81801*y^255
69414*y^254
58754*y^253
49668*y^252
41869*y^251
35251*y^250
29588*y^249
24803*y^248
20722*y^247
17293*y^246
14375*y^245
11937*y^244
9871*y^243
8154*y^242
6703*y^241
5507*y^240
4498*y^239
3673*y^238
2980*y^237
2417*y^236
1946*y^235
1568*y^234
1251*y^233
1000*y^232
791*y^231
627*y^230
490*y^229
385*y^228
297*y^227
231*y^226
176*y^225
135*y^224
101*y^223
77*y^222
56*y^221
42*y^220
30*y^219
22*y^218
15*y^217
11*y^216
7*y^215
5*y^214
3*y^213
2*y^212
1*y^211
1*y^210
the distribution is symmetrical. 810 being the center value (well known to Keno folks)
Last edited by: 7craps on May 20, 2019
winsome johnny (not Win some johnny)
Wizard
Administrator
Wizard
  • Threads: 1493
  • Posts: 26485
Joined: Oct 14, 2009
May 20th, 2019 at 8:08:11 PM permalink
There is blood all over the wall from me trying to make sense of the above post. Please keep in mind I don't have Mathematica and I'm looking for a way to express the combinations in ordinary mathematical language.
"For with much wisdom comes much sorrow." -- Ecclesiastes 1:18 (NIV)
7craps
7craps
  • Threads: 18
  • Posts: 1977
Joined: Jan 23, 2010
May 20th, 2019 at 8:37:37 PM permalink
Quote: Wizard

Please keep in mind I don't have Mathematica

LOL
Mathematica it is not required.
a program that can expand a product function accurately is all that is needed. That is why I use pari/gp as it is free (windows 64 version) and the learning curve is not steep at all if you already know another language.
Quote: Wizard

and I'm looking for a way to express the combinations in ordinary mathematical language.

Don showed why the formula works and it is actually very simple once you understand the concept of a generating function.

I guess I just got lucky as the light switch went on for me. On a scale of 1 to 10 , my math skills are about a 2
your math skills may be way different from mine, but this method of using a generating function is very basic, as generating functions go.

IF you can understand how a GF works with 2d6 and dice sums, you can get this product function.

(x+x^2+x^3+x^4+x^5+x^6)^2. most know how to expand this the long way. we learned this way back in school.
now we have computers that do it for us in 0 ms

you will get it in no time.
winsome johnny (not Win some johnny)
ThatDonGuy
ThatDonGuy
  • Threads: 117
  • Posts: 6267
Joined: Jun 22, 2011
May 20th, 2019 at 8:58:42 PM permalink
Quote: ThatDonGuy

How to get such a count without having to go through all 280 products, I don't know, and presumably, that's why things like Mathematica exist.


I did find this, however, which explains the problem and a way to find the solution
ZPP
ZPP
  • Threads: 2
  • Posts: 31
Joined: Feb 7, 2010
May 21st, 2019 at 12:12:49 AM permalink
Using a computer, this can be calculated using a recurrence:
Let A[n][k][m] be the number of ways to choose k numbers from {1..n} that sum to m.
Suppose we are trying to calculate A[n][k][m] and have already calculated all values for A[n-1][k][m].
We add together the number of ways that do not include n and the number of ways that do include n. This is exhaustive and disjoint.
1. The number of ways that do not include n is A[n-1][k][m], the number of ways to choose k numbers from {1..n-1} that sum to m.
2. If m>=n, the number of ways that do include n is A[n-1][k-1][m-n], the number of ways to choose k-1 numbers from {1..n-1} that sum to m-n. (If m<n, the number of ways to sum to m that do include n is 0.)

This is very fast: using bounds for n, k, and m of 80, 20, and 80*20 (which is overkill, could be 1410), the number of elements to calculate is 80*20*80*20=2,560,000, and each element is at most two array lookups and an addition. It takes less than 10 milliseconds on my laptop.

Here is a C program that does this calculation:
#include <stdio.h>
#include <stdint.h>

#define MAXK 20
#define MAXN 80
#define MAXM (MAXK*MAXN)

// Since 80 choose 20 < 2^64, we can get exact counts with 64-bit ints
// (but note that 80 choose 22 >= 2^64)
typedef uint64_t num_t;
// Just index by size_t
typedef size_t index_t;

// a[n][k][m] is the number of ways to choose k numbers from {1..n} that sum to m
num_t a[MAXN+1][MAXK+1][MAXM+1];
// (Note that because we only reference a[n-1] when calculating a[n],
// we only need two (current and previous) 2-dimensional arrays of size
// MAXK+1 by MAXM+1, not a 3-dimensional array, but nowadays the
// memory needed is nothing.)

int main(void)
{
for (index_t n = 0; n <= MAXN; n++) {
// For any set (including the empty set for n==0), there is 1
// way to choose k==0 numbers and get a sum of 0 and no other
// sum is possible by choosing 0 numbers.
a[n][0][0] = 1;
// Now calculate for choosing more than k>0 numbers.
// We iterate k from 1 to min(n,MAXK).
// Note that therefore we skip this loop if n==0.
index_t maxk = (n < MAXK ? n : MAXK);
for (index_t k = 1; k <= maxk; k++) {
// maxm could be tighter, but this is already fast enough.
index_t maxm = n * k;
for (index_t m = 0; m <= maxm; m++) {
// Calculate a[n][k][m] by adding the number of ways
// that do not include n in the sum and the number of
// ways that do include n in the sum, both of which
// are already calculated in the array.
//
// The number of ways that do not include n is the
// number of ways to choose k numbers from {1..n-1}
// that sum to m.
num_t c = a[n-1][k][m];
// The number of ways that do include n is the number of
// ways to choose k-1 numbers from {1..n-1} that sum
// to m-n, or 0 if m<n.
if (n <= m)
c += a[n-1][k-1][m-n];
a[n][k][m] = c;
}
}
}
// Print results, which are a table listing M and the number of
// ways of choosing MAXK numbers from {1..MAXN} that sum to M
num_t total = 0;
num_t *p = a[MAXN][MAXK];
for (index_t m = 0; m <= MAXM; m++) {
total += p[m];
if (p[m] != 0)
printf("%4zu %19llu\n", m, p[m]);
}
// Total number of ways can be checked to equal MAXN choose MAXK
printf("Sum: %19llu\n", total);
return 0;
}


Here are the results I get:
 210                   1
211 1
212 2
213 3
214 5
215 7
216 11
217 15
218 22
219 30
220 42
221 56
222 77
223 101
224 135
225 176
226 231
227 297
228 385
229 490
230 627
231 791
232 1000
233 1251
234 1568
235 1946
236 2417
237 2980
238 3673
239 4498
240 5507
241 6703
242 8154
243 9871
244 11937
245 14375
246 17293
247 20722
248 24803
249 29588
250 35251
251 41869
252 49668
253 58754
254 69414
255 81801
256 96271
257 113039
258 132559
259 155112
260 181274
261 211428
262 246288
263 286364
264 332557
265 385528
266 446405
267 516054
268 595872
269 686983
270 791131
271 909741
272 1044984
273 1198689
274 1373524
275 1571812
276 1796855
277 2051569
278 2340024
279 2665885
280 3034135
281 3449359
282 3917670
283 4444748
284 5038070
285 5704686
286 6453684
287 7293767
288 8236001
289 9291060
290 10472375
291 11793028
292 13269250
293 14917024
294 16755957
295 18805464
296 21089176
297 23630664
298 26458297
299 29600587
300 33091578
301 36965620
302 41263462
303 46026431
304 51303148
305 57143201
306 63604540
307 70746444
308 78637842
309 87349501
310 96963169
311 107563103
312 119246146
313 132112328
314 146276124
315 161856093
316 178987225
317 197809729
318 218482569
319 241171101
320 266062168
321 293350487
322 323255164
323 356005000
324 391856581
325 431078138
326 473969587
327 520844871
328 572053851
329 627963266
330 688980643
331 755533468
332 828095629
333 907164667
334 993290571
335 1087051347
336 1189084411
337 1300059954
338 1420715489
339 1551826889
340 1694245990
341 1848869496
342 2016679881
343 2198711618
344 2396096082
345 2610024936
346 2841798846
347 3092788314
348 3364486581
349 3658467191
350 3976441876
351 4320214692
352 4691744778
353 5093097335
354 5526511602
355 5994347965
356 6499162145
357 7043648186
358 7630718671
359 8263443950
360 8945138936
361 9679297643
362 10469687646
363 11320279729
364 12235349889
365 13219404469
366 14277290236
367 15414114032
368 16635362300
369 17946814901
370 19354673930
371 20865472184
372 22486212019
373 24224267278
374 26087533643
375 28084323773
376 30223529082
377 32514508546
378 34967262684
379 37592314751
380 40400898786
381 43404832757
382 46616720518
383 50049817550
384 53718247833
385 57636860670
386 61821464449
387 66288674187
388 71056162189
389 76142496873
390 81567411419
391 87351632249
392 93517168095
393 100087127594
394 107086028628
395 114539605867
396 122475141756
397 130921262045
398 139908291043
399 149468034620
400 159634159836
401 170441966142
402 181928790813
403 194133766301
404 207098254570
405 220865589927
406 235481542190
407 250994045973
408 267453694512
409 284913452879
410 303429185857
411 323059354425
412 343865577844
413 365912314318
414 389267459099
415 414002006749
416 440190689221
417 467911618783
418 497246966079
419 528282584833
420 561108732221
421 595819672189
422 632514442353
423 671296435063
424 712274211009
425 755561059108
426 801275859118
427 849542617012
428 900491381529
429 954257753733
430 1010983857544
431 1070817824773
432 1133914822434
433 1200436509276
434 1270552124993
435 1344437917067
436 1422278292091
437 1504265214046
438 1590599420536
439 1681489788194
440 1777154619819
441 1877820975168
442 1983726028916
443 2095116368246
444 2212249424868
445 2335392734382
446 2464825448513
447 2600837554495
448 2743731467669
449 2893821211946
450 3051434096031
451 3216909849508
452 3390602389686
453 3572878910940
454 3764121741811
455 3964727389027
456 4175108488550
457 4395692797816
458 4626925247931
459 4869266881582
460 5123197006289
461 5389212078927
462 5667827963566
463 5959578755669
464 6265019152487
465 6584723213704
466 6919286843610
467 7269326489361
468 7635481738958
469 8018413948848
470 8418808965914
471 8837375680898
472 9274848873719
473 9731987693363
474 10209578630140
475 10708433913262
476 11229394619008
477 11773328981483
478 12341135635235
479 12933741839899
480 13552106861442
481 14197220100303
482 14870104620270
483 15571815175325
484 16303441884554
485 17066108157849
486 17860974520443
487 18689236426307
488 19552128242181
489 20450920942555
490 21386926250737
491 22361494215565
492 23376017513498
493 24431928894154
494 25530705653070
495 26673866939713
496 27862978398196
497 29099649337087
498 30385537541434
499 31722346290416
500 33111829351342
501 34555787837266
502 36056075379039
503 37614594822327
504 39233303581272
505 40914210159126
506 42659379693854
507 44470930294083
508 46351038772509
509 48301936795528
510 50325916807381
511 52425327976532
512 54602582321801
513 56860150445667
514 59200567856702
515 61626430488492
516 64140401221500
517 66745205168528
518 69443636407019
519 72238553020681
520 75132884036461
521 78129624219915
522 81231841219122
523 84442670093739
524 87765320677722
525 91203071830294
526 94759279010024
527 98437368245632
528 102240843924128
529 106173282459017
530 110238340304328
531 114439747308027
532 118781314944316
533 123266929349329
534 127900559773363
535 132686251275763
536 137628133409981
537 142730412563985
538 147997380868998
539 153433408182971
540 159042951225485
541 164830545178892
542 170800813062207
543 176958456936318
544 183308267507535
545 189855114934011
546 196603958660512
547 203559837800120
548 210727881214639
549 218113297456569
550 225721385084601
551 233557522163930
552 241627176819381
553 249935896266269
554 258489317615563
555 267293156421037
556 276353217726061
557 285675384124940
558 295265627053394
559 305129994336046
560 315274621737283
561 325705719979621
562 336429586543371
563 347452592155562
564 358781192841696
565 370421915855964
566 382381372002154
567 394666240991089
568 407283284019903
569 420239328556562
570 433541281180913
571 447196111767398
572 461210866607609
573 475592651976889
574 490348647526558
575 505486089237058
576 521012283085036
577 536934587353256
578 553260426593322
579 569997273279789
580 587152662058258
581 604734170747184
582 622749434876858
583 641206128010489
584 660111976597920
585 679474739604064
586 699302223666421
587 719602262037791
588 740382730055707
589 761651523370470
590 783416573752126
591 805685826594439
592 828467257050350
593 851768846820577
594 875598600626154
595 899964522255079
596 924874631401209
597 950336938960611
598 976359464227661
599 1002950209453776
600 1030117177412699
601 1057868345195045
602 1086211682173956
603 1115155123030642
604 1144706586113291
605 1174873945708909
606 1205665050808215
607 1237087696599984
608 1269149643677458
609 1301858588751637
610 1335222184292444
611 1369248008480895
612 1403943585301204
613 1439316353708705
614 1475373688210195
615 1512122867248405
616 1549571094266055
617 1587725465329523
618 1626592990693830
619 1666180561645293
620 1706494972609528
621 1747542887215360
622 1789330860941288
623 1831865306428571
624 1875152516686548
625 1919198629635734
626 1964009651915660
627 2009591422664545
628 2055949637927477
629 2103089813703016
630 2151017310973098
631 2199737298000372
632 2249254776027773
633 2299574540834782
634 2350701209103646
635 2402639179270463
636 2455392658582214
637 2508965623226933
638 2563361846130743
639 2618584856379097
640 2674637967752948
641 2731524237478957
642 2789246495528881
643 2847807302685929
644 2907208980659856
645 2967453569482805
646 3028542858437463
647 3090478342825046
648 3153261255732853
649 3216892524160984
650 3281372801682291
651 3346702423944313
652 3412881442219846
653 3479909578328192
654 3547786259101073
655 3616510570712255
656 3686081294113980
657 3756496858791478
658 3827755379165355
659 3899854607818599
660 3972791972891010
661 4046564530766019
662 4121169004511073
663 4196601736043217
664 4272858725610066
665 4349935583482269
666 4427827570497986
667 4506529549270567
668 4586036025851501
669 4666341100470600
670 4747438510307985
671 4829321579817147
672 4911983264628415
673 4995416101440403
674 5079612253106599
675 5164563458113973
676 5250261076840903
677 5336696040671980
678 5423858899445150
679 5511739770187560
680 5600328385803359
681 5689614043446161
682 5779585654429600
683 5870231692286645
684 5961540243918360
685 6053498957320960
686 6146095094019128
687 6239315476476153
688 6333146541790366
689 6427574288835366
690 6522584333229145
691 6618161854183309
692 6714291650789160
693 6810958088586958
694 6908145157139459
695 7005836416368480
696 7104015055421622
697 7202663838749386
698 7301765166303075
699 7401301019361722
700 7501253022027266
701 7601602386843987
702 7702329977589080
703 7803416254655222
704 7904841339170873
705 8006584958150181
706 8108626509899889
707 8210945008978195
708 8313519152881939
709 8416327266778085
710 8519347371498567
711 8622557128046053
712 8725933906847840
713 8829454732072484
714 8933096352127635
715 9036835183748460
716 9140647383762528
717 9244508792945678
718 9348395008996007
719 9452281330191897
720 9556142829555643
721 9659954298267308
722 9763690321033408
723 9867325219249981
724 9970833127512512
725 10074187936558122
726 10177363370888455
727 10280332931441733
728 10383069974340297
729 10485547653279647
730 10587738999328217
731 10689616863061737
732 10791153995382729
733 10892322989344543
734 10993096361994634
735 11093446495874170
736 11193345721807668
737 11292766260101878
738 11391680304241666
739 11490059961730183
740 11587877338692722
741 11685104480338701
742 11781713456388257
743 11877676301179640
744 11972965099875122
745 12067551928151070
746 12161408939178544
747 12254508302876326
748 12346822293578054
749 12438323228868060
750 12528983557886059
751 12618775799682379
752 12707672632154594
753 12795646829899064
754 12882671353685064
755 12968719287824486
756 13053763930133222
757 13137778728752338
758 13220737372587114
759 13302613727557896
760 13383381927422176
761 13463016309475063
762 13541491505700427
763 13618782377856223
764 13694864108945970
765 13769712137666260
766 13843302250100143
767 13915610513550433
768 13986613368402674
769 14056287561278035
770 14124610237055954
771 14191558871327486
772 14257111362481292
773 14321245963481667
774 14383941373963924
775 14445176671275633
776 14504931402574878
777 14563185515116493
778 14619919448256126
779 14675114063011110
780 14728750733921081
781 14780811277822859
782 14831278045561927
783 14880133849969280
784 14927362057331041
785 14972946514599961
786 15016871640574771
787 15059122352292240
788 15099684155916740
789 15138543072303315
790 15175685727506919
791 15211099277556439
792 15244771498541818
793 15276690710548182
794 15306845867323822
795 15335226479373283
796 15361822703122288
797 15386625263213894
798 15409625541131887
799 15430815496656959
800 15450187755959996
801 15467735532221058
802 15483452713112335
803 15497333780632956
804 15509373897951036
805 15519568828416239
806 15527915021776296
807 15534409532379142
808 15539050104692302
809 15541835090752144
810 15542763534960598
811 15541835090752144
812 15539050104692302
813 15534409532379142
814 15527915021776296
815 15519568828416239
816 15509373897951036
817 15497333780632956
818 15483452713112335
819 15467735532221058
820 15450187755959996
821 15430815496656959
822 15409625541131887
823 15386625263213894
824 15361822703122288
825 15335226479373283
826 15306845867323822
827 15276690710548182
828 15244771498541818
829 15211099277556439
830 15175685727506919
831 15138543072303315
832 15099684155916740
833 15059122352292240
834 15016871640574771
835 14972946514599961
836 14927362057331041
837 14880133849969280
838 14831278045561927
839 14780811277822859
840 14728750733921081
841 14675114063011110
842 14619919448256126
843 14563185515116493
844 14504931402574878
845 14445176671275633
846 14383941373963924
847 14321245963481667
848 14257111362481292
849 14191558871327486
850 14124610237055954
851 14056287561278035
852 13986613368402674
853 13915610513550433
854 13843302250100143
855 13769712137666260
856 13694864108945970
857 13618782377856223
858 13541491505700427
859 13463016309475063
860 13383381927422176
861 13302613727557896
862 13220737372587114
863 13137778728752338
864 13053763930133222
865 12968719287824486
866 12882671353685064
867 12795646829899064
868 12707672632154594
869 12618775799682379
870 12528983557886059
871 12438323228868060
872 12346822293578054
873 12254508302876326
874 12161408939178544
875 12067551928151070
876 11972965099875122
877 11877676301179640
878 11781713456388257
879 11685104480338701
880 11587877338692722
881 11490059961730183
882 11391680304241666
883 11292766260101878
884 11193345721807668
885 11093446495874170
886 10993096361994634
887 10892322989344543
888 10791153995382729
889 10689616863061737
890 10587738999328217
891 10485547653279647
892 10383069974340297
893 10280332931441733
894 10177363370888455
895 10074187936558122
896 9970833127512512
897 9867325219249981
898 9763690321033408
899 9659954298267308
900 9556142829555643
901 9452281330191897
902 9348395008996007
903 9244508792945678
904 9140647383762528
905 9036835183748460
906 8933096352127635
907 8829454732072484
908 8725933906847840
909 8622557128046053
910 8519347371498567
911 8416327266778085
912 8313519152881939
913 8210945008978195
914 8108626509899889
915 8006584958150181
916 7904841339170873
917 7803416254655222
918 7702329977589080
919 7601602386843987
920 7501253022027266
921 7401301019361722
922 7301765166303075
923 7202663838749386
924 7104015055421622
925 7005836416368480
926 6908145157139459
927 6810958088586958
928 6714291650789160
929 6618161854183309
930 6522584333229145
931 6427574288835366
932 6333146541790366
933 6239315476476153
934 6146095094019128
935 6053498957320960
936 5961540243918360
937 5870231692286645
938 5779585654429600
939 5689614043446161
940 5600328385803359
941 5511739770187560
942 5423858899445150
943 5336696040671980
944 5250261076840903
945 5164563458113973
946 5079612253106599
947 4995416101440403
948 4911983264628415
949 4829321579817147
950 4747438510307985
951 4666341100470600
952 4586036025851501
953 4506529549270567
954 4427827570497986
955 4349935583482269
956 4272858725610066
957 4196601736043217
958 4121169004511073
959 4046564530766019
960 3972791972891010
961 3899854607818599
962 3827755379165355
963 3756496858791478
964 3686081294113980
965 3616510570712255
966 3547786259101073
967 3479909578328192
968 3412881442219846
969 3346702423944313
970 3281372801682291
971 3216892524160984
972 3153261255732853
973 3090478342825046
974 3028542858437463
975 2967453569482805
976 2907208980659856
977 2847807302685929
978 2789246495528881
979 2731524237478957
980 2674637967752948
981 2618584856379097
982 2563361846130743
983 2508965623226933
984 2455392658582214
985 2402639179270463
986 2350701209103646
987 2299574540834782
988 2249254776027773
989 2199737298000372
990 2151017310973098
991 2103089813703016
992 2055949637927477
993 2009591422664545
994 1964009651915660
995 1919198629635734
996 1875152516686548
997 1831865306428571
998 1789330860941288
999 1747542887215360
1000 1706494972609528
1001 1666180561645293
1002 1626592990693830
1003 1587725465329523
1004 1549571094266055
1005 1512122867248405
1006 1475373688210195
1007 1439316353708705
1008 1403943585301204
1009 1369248008480895
1010 1335222184292444
1011 1301858588751637
1012 1269149643677458
1013 1237087696599984
1014 1205665050808215
1015 1174873945708909
1016 1144706586113291
1017 1115155123030642
1018 1086211682173956
1019 1057868345195045
1020 1030117177412699
1021 1002950209453776
1022 976359464227661
1023 950336938960611
1024 924874631401209
1025 899964522255079
1026 875598600626154
1027 851768846820577
1028 828467257050350
1029 805685826594439
1030 783416573752126
1031 761651523370470
1032 740382730055707
1033 719602262037791
1034 699302223666421
1035 679474739604064
1036 660111976597920
1037 641206128010489
1038 622749434876858
1039 604734170747184
1040 587152662058258
1041 569997273279789
1042 553260426593322
1043 536934587353256
1044 521012283085036
1045 505486089237058
1046 490348647526558
1047 475592651976889
1048 461210866607609
1049 447196111767398
1050 433541281180913
1051 420239328556562
1052 407283284019903
1053 394666240991089
1054 382381372002154
1055 370421915855964
1056 358781192841696
1057 347452592155562
1058 336429586543371
1059 325705719979621
1060 315274621737283
1061 305129994336046
1062 295265627053394
1063 285675384124940
1064 276353217726061
1065 267293156421037
1066 258489317615563
1067 249935896266269
1068 241627176819381
1069 233557522163930
1070 225721385084601
1071 218113297456569
1072 210727881214639
1073 203559837800120
1074 196603958660512
1075 189855114934011
1076 183308267507535
1077 176958456936318
1078 170800813062207
1079 164830545178892
1080 159042951225485
1081 153433408182971
1082 147997380868998
1083 142730412563985
1084 137628133409981
1085 132686251275763
1086 127900559773363
1087 123266929349329
1088 118781314944316
1089 114439747308027
1090 110238340304328
1091 106173282459017
1092 102240843924128
1093 98437368245632
1094 94759279010024
1095 91203071830294
1096 87765320677722
1097 84442670093739
1098 81231841219122
1099 78129624219915
1100 75132884036461
1101 72238553020681
1102 69443636407019
1103 66745205168528
1104 64140401221500
1105 61626430488492
1106 59200567856702
1107 56860150445667
1108 54602582321801
1109 52425327976532
1110 50325916807381
1111 48301936795528
1112 46351038772509
1113 44470930294083
1114 42659379693854
1115 40914210159126
1116 39233303581272
1117 37614594822327
1118 36056075379039
1119 34555787837266
1120 33111829351342
1121 31722346290416
1122 30385537541434
1123 29099649337087
1124 27862978398196
1125 26673866939713
1126 25530705653070
1127 24431928894154
1128 23376017513498
1129 22361494215565
1130 21386926250737
1131 20450920942555
1132 19552128242181
1133 18689236426307
1134 17860974520443
1135 17066108157849
1136 16303441884554
1137 15571815175325
1138 14870104620270
1139 14197220100303
1140 13552106861442
1141 12933741839899
1142 12341135635235
1143 11773328981483
1144 11229394619008
1145 10708433913262
1146 10209578630140
1147 9731987693363
1148 9274848873719
1149 8837375680898
1150 8418808965914
1151 8018413948848
1152 7635481738958
1153 7269326489361
1154 6919286843610
1155 6584723213704
1156 6265019152487
1157 5959578755669
1158 5667827963566
1159 5389212078927
1160 5123197006289
1161 4869266881582
1162 4626925247931
1163 4395692797816
1164 4175108488550
1165 3964727389027
1166 3764121741811
1167 3572878910940
1168 3390602389686
1169 3216909849508
1170 3051434096031
1171 2893821211946
1172 2743731467669
1173 2600837554495
1174 2464825448513
1175 2335392734382
1176 2212249424868
1177 2095116368246
1178 1983726028916
1179 1877820975168
1180 1777154619819
1181 1681489788194
1182 1590599420536
1183 1504265214046
1184 1422278292091
1185 1344437917067
1186 1270552124993
1187 1200436509276
1188 1133914822434
1189 1070817824773
1190 1010983857544
1191 954257753733
1192 900491381529
1193 849542617012
1194 801275859118
1195 755561059108
1196 712274211009
1197 671296435063
1198 632514442353
1199 595819672189
1200 561108732221
1201 528282584833
1202 497246966079
1203 467911618783
1204 440190689221
1205 414002006749
1206 389267459099
1207 365912314318
1208 343865577844
1209 323059354425
1210 303429185857
1211 284913452879
1212 267453694512
1213 250994045973
1214 235481542190
1215 220865589927
1216 207098254570
1217 194133766301
1218 181928790813
1219 170441966142
1220 159634159836
1221 149468034620
1222 139908291043
1223 130921262045
1224 122475141756
1225 114539605867
1226 107086028628
1227 100087127594
1228 93517168095
1229 87351632249
1230 81567411419
1231 76142496873
1232 71056162189
1233 66288674187
1234 61821464449
1235 57636860670
1236 53718247833
1237 50049817550
1238 46616720518
1239 43404832757
1240 40400898786
1241 37592314751
1242 34967262684
1243 32514508546
1244 30223529082
1245 28084323773
1246 26087533643
1247 24224267278
1248 22486212019
1249 20865472184
1250 19354673930
1251 17946814901
1252 16635362300
1253 15414114032
1254 14277290236
1255 13219404469
1256 12235349889
1257 11320279729
1258 10469687646
1259 9679297643
1260 8945138936
1261 8263443950
1262 7630718671
1263 7043648186
1264 6499162145
1265 5994347965
1266 5526511602
1267 5093097335
1268 4691744778
1269 4320214692
1270 3976441876
1271 3658467191
1272 3364486581
1273 3092788314
1274 2841798846
1275 2610024936
1276 2396096082
1277 2198711618
1278 2016679881
1279 1848869496
1280 1694245990
1281 1551826889
1282 1420715489
1283 1300059954
1284 1189084411
1285 1087051347
1286 993290571
1287 907164667
1288 828095629
1289 755533468
1290 688980643
1291 627963266
1292 572053851
1293 520844871
1294 473969587
1295 431078138
1296 391856581
1297 356005000
1298 323255164
1299 293350487
1300 266062168
1301 241171101
1302 218482569
1303 197809729
1304 178987225
1305 161856093
1306 146276124
1307 132112328
1308 119246146
1309 107563103
1310 96963169
1311 87349501
1312 78637842
1313 70746444
1314 63604540
1315 57143201
1316 51303148
1317 46026431
1318 41263462
1319 36965620
1320 33091578
1321 29600587
1322 26458297
1323 23630664
1324 21089176
1325 18805464
1326 16755957
1327 14917024
1328 13269250
1329 11793028
1330 10472375
1331 9291060
1332 8236001
1333 7293767
1334 6453684
1335 5704686
1336 5038070
1337 4444748
1338 3917670
1339 3449359
1340 3034135
1341 2665885
1342 2340024
1343 2051569
1344 1796855
1345 1571812
1346 1373524
1347 1198689
1348 1044984
1349 909741
1350 791131
1351 686983
1352 595872
1353 516054
1354 446405
1355 385528
1356 332557
1357 286364
1358 246288
1359 211428
1360 181274
1361 155112
1362 132559
1363 113039
1364 96271
1365 81801
1366 69414
1367 58754
1368 49668
1369 41869
1370 35251
1371 29588
1372 24803
1373 20722
1374 17293
1375 14375
1376 11937
1377 9871
1378 8154
1379 6703
1380 5507
1381 4498
1382 3673
1383 2980
1384 2417
1385 1946
1386 1568
1387 1251
1388 1000
1389 791
1390 627
1391 490
1392 385
1393 297
1394 231
1395 176
1396 135
1397 101
1398 77
1399 56
1400 42
1401 30
1402 22
1403 15
1404 11
1405 7
1406 5
1407 3
1408 2
1409 1
1410 1
Sum: 3535316142212174320

Comments:

There is a direct correspondence between this method and the method others have posted of expanding a very large polynomial and looking at certain coefficients. But I wrote this code before realizing how direct the correspondence was, and also no one else has really explained how an efficient calculation of the coefficients works, so here we are.

With regard to the part of the comment by ThatDonGuy specifically about looking at all 280 products, this is not necessary because the largest term, expanding the full polynomial that includes sums of more the 20 balls, is x80y1410. If you collect like terms, there are "only" 81*1411=114291 terms in the full product. If you collect like terms at each stage (each multiplication by (1+xyn)), the number of terms remains manageable at each stage.

However, I think ThatDonGuy has a great explanation of why the polynomial expansion works and I have nothing to add to it. (I just have an explanation of how to efficiently do this calculation.)
weezrDASvegas
weezrDASvegas
  • Threads: 2
  • Posts: 69
Joined: Feb 2, 2018
May 21st, 2019 at 2:25:23 AM permalink
Quote: Wizard

I am doing a software review of a company that offers prop bets on the sum of the balls in keno. As a reminder, in keno the game draws 20 numbers from a 80 balls numbered 1 to 80. The problem at hand is finding a probability of any given total without using a simulation.

I do know simulations are a perfectly valid way of analyzing casino games, but I also feel more satisfied finding an exact answer. At first I wrote a simple program to loop through 3,535,316,142,212,180,000 possible ways to choose 20 out of 80 balls. Needless to say, it would have taken centuries to cycle through them all. I've also toyed with shortcut code, but I fear I'll die before the program is done running.

I've also just counted combinations by hand, hoping to find a pattern (maybe some variant of Fibonaci), but so far have not found anything. So, at this point I open it up to the forum. You do not have to put replies in spoiler tags and may refer to any outside sources. This isn't the usual math puzzle thread.

The question for the poll is what are your thoughts on it?

In this spoiler tag are the results of my simulation. I excluded extreme totals where the probability was less than 1 in a million. It's a big table, so I won't clutter up the thread with it, unless you want to see it. The third column is the expected count per million games.




Aint a formula for that. gotta do it in soft code - iterate thru all combos in the game.

There is a website that shows the sums for many lottery games but no keno. probably keno has too many combos compared 2 lottos, even mega mills or powerball.

Total powerball combos (5 of 69 & 1 26)=292,201,338
Total keno combos (20 from 80)=3.53531614221217E+18 (19 digit number... very, very large for the computers).

But looks like the right algo exists and applicable to any lotto game.

https://saliu.com/forum/lottery-sums.html

Powerball 5/69 & 1/26 Lottery SUMS Chart



Sum-total Combinations Percentage

16 1 0.00%
17 2 0.00%
18 4 0.00%
19 7 0.00%
20 12 0.00%
21 19 0.00%
22 29 0.00%
23 42 0.00%
24 60 0.00%
25 83 0.00%
26 113 0.00%
27 150 0.00%
28 197 0.00%
29 254 0.00%
30 324 0.00%
31 408 0.00%
32 509 0.00%
33 628 0.00%
34 769 0.00%
35 933 0.00%
36 1125 0.00%
37 1346 0.00%
38 1601 0.00%
39 1892 0.00%
40 2225 0.00%
41 2602 0.00%
42 3028 0.00%
43 3507 0.00%
44 4045 0.00%
45 4645 0.00%
46 5314 0.00%
47 6055 0.00%
48 6876 0.00%
49 7781 0.00%
50 8777 0.00%
51 9869 0.00%
52 11065 0.00%
53 12370 0.00%
54 13792 0.00%
55 15337 0.01%
56 17014 0.01%
57 18828 0.01%
58 20789 0.01%
59 22903 0.01%
60 25180 0.01%
61 27627 0.01%
62 30253 0.01%
63 33066 0.01%
64 36077 0.01%
65 39293 0.01%
66 42725 0.01%
67 46381 0.02%
68 50273 0.02%
69 54409 0.02%
70 58801 0.02%
71 63458 0.02%
72 68392 0.02%
73 73613 0.03%
74 79133 0.03%
75 84962 0.03%
76 91114 0.03%
77 97598 0.03%
78 104428 0.04%
79 111615 0.04%
80 119173 0.04%
81 127112 0.04%
82 135446 0.05%
83 144186 0.05%
84 153346 0.05%
85 162936 0.06%
86 172970 0.06%
87 183457 0.06%
88 194412 0.07%
89 205843 0.07%
90 217763 0.07%
91 230181 0.08%
92 243109 0.08%
93 256554 0.09%
94 270528 0.09%
95 285037 0.10%
96 300092 0.10%
97 315697 0.11%
98 331862 0.11%
99 348590 0.12%
100 365890 0.13%
101 383763 0.13%
102 402215 0.14%
103 421247 0.14%
104 440864 0.15%
105 461063 0.16%
106 481848 0.16%
107 503215 0.17%
108 525166 0.18%
109 547695 0.19%
110 570802 0.20%
111 594480 0.20%
112 618727 0.21%
113 643534 0.22%
114 668897 0.23%
115 694806 0.24%
116 721256 0.25%
117 748233 0.26%
118 775731 0.27%
119 803736 0.28%
120 832239 0.28%
121 861224 0.29%
122 890680 0.30%
123 920590 0.32%
124 950942 0.33%
125 981716 0.34%
126 1012898 0.35%
127 1044467 0.36%
128 1076408 0.37%
129 1108697 0.38%
130 1141317 0.39%
131 1174244 0.40%
132 1207458 0.41%
133 1240933 0.42%
134 1274648 0.44%
135 1308575 0.45%
136 1342692 0.46%
137 1376968 0.47%
138 1411379 0.48%
139 1445894 0.49%
140 1480487 0.51%
141 1515124 0.52%
142 1549777 0.53%
143 1584412 0.54%
144 1618999 0.55%
145 1653501 0.57%
146 1687887 0.58%
147 1722119 0.59%
148 1756165 0.60%
149 1789986 0.61%
150 1823548 0.62%
151 1856813 0.64%
152 1889746 0.65%
153 1922308 0.66%
154 1954464 0.67%
155 1986176 0.68%
156 2017409 0.69%
157 2048124 0.70%
158 2078287 0.71%
159 2107861 0.72%
160 2136812 0.73%
161 2165104 0.74%
162 2192703 0.75%
163 2219576 0.76%
164 2245691 0.77%
165 2271015 0.78%
166 2295518 0.79%
167 2319170 0.79%
168 2341943 0.80%
169 2363809 0.81%
170 2384742 0.82%
171 2404718 0.82%
172 2423713 0.83%
173 2441705 0.84%
174 2458673 0.84%
175 2474598 0.85%
176 2489462 0.85%
177 2503247 0.86%
178 2515938 0.86%
179 2527521 0.86%
180 2537983 0.87%
181 2547312 0.87%
182 2555497 0.87%
183 2562530 0.88%
184 2568403 0.88%
185 2573109 0.88%
186 2576643 0.88%
187 2579001 0.88%

188 2580181 0.88%
189 2580181 0.88%

190 2579001 0.88%
191 2576643 0.88%
192 2573109 0.88%
193 2568403 0.88%
194 2562530 0.88%
195 2555497 0.87%
196 2547312 0.87%
197 2537983 0.87%
198 2527521 0.86%
199 2515938 0.86%
200 2503247 0.86%
201 2489462 0.85%
202 2474598 0.85%
203 2458673 0.84%
204 2441705 0.84%
205 2423713 0.83%
206 2404718 0.82%
207 2384742 0.82%
208 2363809 0.81%
209 2341943 0.80%
210 2319170 0.79%
211 2295518 0.79%
212 2271015 0.78%
213 2245691 0.77%
214 2219576 0.76%
215 2192703 0.75%
216 2165104 0.74%
217 2136812 0.73%
218 2107861 0.72%
219 2078287 0.71%
220 2048124 0.70%
221 2017409 0.69%
222 1986176 0.68%
223 1954464 0.67%
224 1922308 0.66%
225 1889746 0.65%
226 1856813 0.64%
227 1823548 0.62%
228 1789986 0.61%
229 1756165 0.60%
230 1722119 0.59%
231 1687887 0.58%
232 1653501 0.57%
233 1618999 0.55%
234 1584412 0.54%
235 1549777 0.53%
236 1515124 0.52%
237 1480487 0.51%
238 1445894 0.49%
239 1411379 0.48%
240 1376968 0.47%
241 1342692 0.46%
242 1308575 0.45%
243 1274648 0.44%
244 1240933 0.42%
245 1207458 0.41%
246 1174244 0.40%
247 1141317 0.39%
248 1108697 0.38%
249 1076408 0.37%
250 1044467 0.36%
251 1012898 0.35%
252 981716 0.34%
253 950942 0.33%
254 920590 0.32%
255 890680 0.30%
256 861224 0.29%
257 832239 0.28%
258 803736 0.28%
259 775731 0.27%
260 748233 0.26%
261 721256 0.25%
262 694806 0.24%
263 668897 0.23%
264 643534 0.22%
265 618727 0.21%
266 594480 0.20%
267 570802 0.20%
268 547695 0.19%
269 525166 0.18%
270 503215 0.17%
271 481848 0.16%
272 461063 0.16%
273 440864 0.15%
274 421247 0.14%
275 402215 0.14%
276 383763 0.13%
277 365890 0.13%
278 348590 0.12%
279 331862 0.11%
280 315697 0.11%
281 300092 0.10%
282 285037 0.10%
283 270528 0.09%
284 256554 0.09%
285 243109 0.08%
286 230181 0.08%
287 217763 0.07%
288 205843 0.07%
289 194412 0.07%
290 183457 0.06%
291 172970 0.06%
292 162936 0.06%
293 153346 0.05%
294 144186 0.05%
295 135446 0.05%
296 127112 0.04%
297 119173 0.04%
298 111615 0.04%
299 104428 0.04%
300 97598 0.03%
301 91114 0.03%
302 84962 0.03%
303 79133 0.03%
304 73613 0.03%
305 68392 0.02%
306 63458 0.02%
307 58801 0.02%
308 54409 0.02%
309 50273 0.02%
310 46381 0.02%
311 42725 0.01%
312 39293 0.01%
313 36077 0.01%
314 33066 0.01%
315 30253 0.01%
316 27627 0.01%
317 25180 0.01%
318 22903 0.01%
319 20789 0.01%
320 18828 0.01%
321 17014 0.01%
322 15337 0.01%
323 13792 0.00%
324 12370 0.00%
325 11065 0.00%
326 9869 0.00%
327 8777 0.00%
328 7781 0.00%
329 6876 0.00%
330 6055 0.00%
331 5314 0.00%
332 4645 0.00%
333 4045 0.00%
334 3507 0.00%
335 3028 0.00%
336 2602 0.00%
337 2225 0.00%
338 1892 0.00%
339 1601 0.00%
340 1346 0.00%
341 1125 0.00%
342 933 0.00%
343 769 0.00%
344 628 0.00%
345 509 0.00%
346 408 0.00%
347 324 0.00%
348 254 0.00%
349 197 0.00%
350 150 0.00%
351 113 0.00%
352 83 0.00%
353 60 0.00%
354 42 0.00%
355 29 0.00%
356 19 0.00%
357 12 0.00%
358 7 0.00%
359 4 0.00%
360 2 0.00%
361 1 0.00%

Total: 346 292201338 100%


Quote: Wizard

Note: Post formatting corrected by management.

Last edited by: unnamed administrator on May 21, 2019
Wizard
Administrator
Wizard
  • Threads: 1493
  • Posts: 26485
Joined: Oct 14, 2009
May 21st, 2019 at 6:40:07 AM permalink
I get the theory about finding the generating function, but to actually do it would be the equal amount of work as just cycling through all combin(80,20) balls by brute force.

ZPP, thanks for the code, I think I will try that method next. I started to go down that road, but didn't finishing, thinking it would only cut down centuries of computer time to years. Let's see what happens....
"For with much wisdom comes much sorrow." -- Ecclesiastes 1:18 (NIV)
7craps
7craps
  • Threads: 18
  • Posts: 1977
Joined: Jan 23, 2010
May 21st, 2019 at 7:28:40 AM permalink
Quote: Wizard

I get the theory about finding the generating function, but to actually do it would be the equal amount of work as just cycling through all combin(80,20) balls by brute force.

the generating function is also easily changed for the values on the balls. say instead of 1 to 80 they were 0 to 79. simple adjustment

it takes 0ms on my 10 year old Dell 2 core laptop for all the calculations (according to pari/gp calculator) and it is actually doing all the draws from 0 to 80 inclusive and prints to a text file.

from there it is easy (for me) to get what is needed into Excel or work on the results further in pari.
generating functions go way back and it makes sense why.(they had no computers to do it then)
even poker gurus now (like BA) uses GFs to describe getting particular hands and so on.

of course there are 'expert computer guys and gals' that can do excellent code for almost any problem.

some get (and can use) generating functions and add them to their toolbox, others never get them for whatever reason(s).

added: reminds me of learning inclusion-exclusion and wondering why we have to do it this way. add and subtract over-counts seemed way confusing and not required as they were always another way to solve the problem. Most now, that even do not understand PIE, still use it in a computer to solve some problems because it IS so easy and follows basic counting methods. example code for coupon collecting problem: n=6;\\number of coupons
t=10;\\number of draws
x=sum(k=0,n,(-1)^k*binomial(n,k)*(1-(k/n))^t);

this was so confusing at 1st: (-1)^k*binomial(n,k)*(1-(k/n))^t
Last edited by: 7craps on May 21, 2019
winsome johnny (not Win some johnny)
charliepatrick
charliepatrick
  • Threads: 39
  • Posts: 2946
Joined: Jun 17, 2011
Thanked by
RS
May 21st, 2019 at 7:34:51 AM permalink
I'm about to go out (it's afternoon shoping here) but here's an idea I might look at later.
(i) Split the numbers into blocks 1-10, 11-20 etc. (consider 1-10 as 0*10+1 thru 0*10+10, 2-20 at 1*10+1 thru 1*10+10...)
(ii) Similar to working out poker hands, you can have a pattern of (a) 10 in one block, 10 in another (b) 10 9 1 (c) 10 8 2 (d) 10 8 1 1 etc.
(iii) Work out the permutations for various numbers of balls within a block for the digits. (e..g 1 number in block can be 1:1 2:1 ... 10:1).
(iv) Work out the permutations for various blocks being used, this will only affect the tens value of the total.
(v) Add together the digits part from (iii) and the tens part from (iv).
e.g. Looking at {10,10}, the sum of the digits part is fixed at 55+55=110. The sum of the tens part can be 10*0+10*10, 10*0+10*20, 10*10+10*10 etc. This gets 1 combination for 210 etc.
e.g. Looking at (10,9,1), the sum of the digits part can be {55}+{45,47,48,49...54)+(1,2,3,4,...,9,10}. Then the tens can be 10*0+9*10+1*20 etc.
gordonm888
Administrator
gordonm888
  • Threads: 60
  • Posts: 5043
Joined: Feb 18, 2015
May 21st, 2019 at 10:21:34 AM permalink
Quote: charliepatrick

I'm about to go out (it's afternoon shoping here) but here's an idea I might look at later.
(i) Split the numbers into blocks 1-10, 11-20 etc. (consider 1-10 as 0*10+1 thru 0*10+10, 2-20 at 1*10+1 thru 1*10+10...)
(ii) Similar to working out poker hands, you can have a pattern of (a) 10 in one block, 10 in another (b) 10 9 1 (c) 10 8 2 (d) 10 8 1 1 etc.
(iii) Work out the permutations for various numbers of balls within a block for the digits. (e..g 1 number in block can be 1:1 2:1 ... 10:1).
(iv) Work out the permutations for various blocks being used, this will only affect the tens value of the total.
(v) Add together the digits part from (iii) and the tens part from (iv).
e.g. Looking at {10,10}, the sum of the digits part is fixed at 55+55=110. The sum of the tens part can be 10*0+10*10, 10*0+10*20, 10*10+10*10 etc. This gets 1 combination for 210 etc.
e.g. Looking at (10,9,1), the sum of the digits part can be {55}+{45,47,48,49...54)+(1,2,3,4,...,9,10}. Then the tens can be 10*0+9*10+1*20 etc.



I like this approach, ingenious. I wonder whether it might be slightly more efficient in base 9.

edit: Or 4-tier it in base 3.
Last edited by: gordonm888 on May 21, 2019
So many better men, a few of them friends, are dead. And a thousand thousand slimy things live on, and so do I.
Wizard
Administrator
Wizard
  • Threads: 1493
  • Posts: 26485
Joined: Oct 14, 2009
May 21st, 2019 at 2:29:50 PM permalink
I didn't get very far with the code above. My C++ compiler didn't like this statement and I don't understand what it is supposed to:

typedef uint64_t num_t;

However, I can't ask anyone to teach me a new computer language. I was half finished doing it this way anyway, so will try to complete it with my own code.

Thanks ZPP for your help!
"For with much wisdom comes much sorrow." -- Ecclesiastes 1:18 (NIV)
ThatDonGuy
ThatDonGuy
  • Threads: 117
  • Posts: 6267
Joined: Jun 22, 2011
May 21st, 2019 at 3:43:53 PM permalink
Quote: ThatDonGuy

I think I sort of do.
The formula is the product of:
1 + x y
1 + x y2
1 + x y3
1 + x y4
...
1 + x y79
1 + x y80

If you draw 20 particular balls, multiply the product of the left sides of the terms of the balls you did not draw and the right sides of the balls you did draw together; you will get x20 and ysum of the numbers.
To determine how many times 20 balls add up to some number N, count the number of times x20 yN is such a product.

How to get such a count without having to go through all 280 products, I don't know, and presumably, that's why things like Mathematica exist.


HOUSE LIGHTBULB MOMENT - this may be similar to ZPP's solution

Instead of treating those as 80 different sums, do the products from top to bottom - i.e.
Start with (1 + xy) (1 + x y2)
Multiply that by (1 + x y3)
Multiply that result by (1 + x y4)
and so on through (1 + x y80)

This can be done with an array, where each row is one of the y exponents (from 0 to 1410) and each column is one of the x exponents (from 0 to 20)
Cell (a, b) is the current number of times xa yb appears in the solution so far
Start with all cells 0, except for 1s in (0,0) and (1,1); this is 1 + xy
Each step multiplies the existing value by x yn n for some n from 1 to 80, then adding it to the existing result. This is done by making a copy of the array, shifting it one column to the right and n rows down, and adding those values to the existing values.
Once all 80 steps are done, cell (20, N) will contain the number of combinations of 20 balls that add up to N.
This took 1/2 second to calculate in C#.
rsactuary
rsactuary
  • Threads: 29
  • Posts: 2315
Joined: Sep 6, 2014
May 21st, 2019 at 6:36:32 PM permalink
In your WOO post, you don't define rain, cloud, lightning, fire and sprout.

I find it odd that small vs big has a different payout, given the symmetrical distribution. Any sense as to why they might do that?
Wizard
Administrator
Wizard
  • Threads: 1493
  • Posts: 26485
Joined: Oct 14, 2009
May 21st, 2019 at 7:19:41 PM permalink
Quote: rsactuary

In your WOO post, you don't define rain, cloud, lightning, fire and sprout.

I find it odd that small vs big has a different payout, given the symmetrical distribution. Any sense as to why they might do that?



Good catches, thanks. The big had a typo in the win. I just added a description of the ranges covered by the bets you asked about.

For everyone else, here is my new page on the game with the prop bets on the total off all balls: Blitz Bingo.
"For with much wisdom comes much sorrow." -- Ecclesiastes 1:18 (NIV)
7craps
7craps
  • Threads: 18
  • Posts: 1977
Joined: Jan 23, 2010
May 21st, 2019 at 7:36:37 PM permalink
Quote: Wizard

For everyone else, here is my new page on the game with the prop bets on the total off all balls: Blitz Bingo.

3,535,316,142,212,170,000
should be
3,535,316,142,212,174,320
Combin(80,20) on Wolfram Alpha
winsome johnny (not Win some johnny)
DogHand
DogHand 
  • Threads: 2
  • Posts: 1517
Joined: Sep 24, 2011
May 21st, 2019 at 8:37:45 PM permalink
Wizard,

Rule 5 has a typo on the payout for Big: it says 1.91, but on your EV table you have 1.95.

Dog Hand
Wizard
Administrator
Wizard
  • Threads: 1493
  • Posts: 26485
Joined: Oct 14, 2009
May 21st, 2019 at 9:15:09 PM permalink
I'm happy to report my program worked and it runs in under a second. I'll include the code here, not that anyone will care.


#include <iostream>
#include <stdlib.h>
using namespace std;


void fast(void);



__int64 a[81][21][1601];

int main(void)
{
int i;
fast();
cerr << "Enter any number\t";
cin >> i;
return 1;
}


void fast(void)
{
int k,m,n,t;

for (k=0; k<=80; k++) // lowest available ball
{
for (m=0; m<=20; m++)
{
for (n=0; n<=1600; n++)
a[k][m][n]=0;
}
}
for (k=20; k<=80; k++)
{
for (m=k; m<=80; m++) // 20th ball
a[k][20][m]=1;
}
for (k=19; k<=79; k++) // lowest available ball
{
for (m=k; m<=79; m++) // 19th ball
{
for (t=0; t<=80; t++)
{
a[k][19][m+t]+=a[m+1][20][t];
}
}
}
for (k=18; k<=78; k++) // lowest available ball
{
for (m=k; m<=78; m++) // 18th ball
{
for (t=0; t<=160; t++)
{
a[k][18][m+t]+=a[m+1][19][t];
}
}
}
int level;
for (level=17; level>=1; level--)
{
for (k=level; k<=60+level; k++) // lowest available ball
{
for (m=k; m<=60+level; m++) // level-th ball
{
for (t=0; t<=1410; t++)
{
a[k][level][m+t]+=a[m+1][level+1][t];
}
}
}
}

printf("Raw totals\n");
for (n=0; n<=1410; n++)
{
if (a[1][1][n]>0)
printf("%i\t%I64i\n",n,a[1][1][n]);
}

printf("formatted totals\n");
for (n=0; n<=1410; n++)
{
if (a[1][1][n]>0)
printf("<tr><td>%i</td><td>%I64i</td></tr>\n",n,a[1][1][n]);
}
return;
}


I'd like to thank all who participated! This a great example of why I love this forum. I hope others have learned something as well.


Quote: 7craps

Quote: Wizard

For everyone else, here is my new page on the game with the prop bets on the total off all balls: Blitz Bingo.

3,535,316,142,212,170,000
should be
3,535,316,142,212,174,320
Combin(80,20) on Wolfram Alpha



Damn Microsoft! Excel allows for only so many significant digits (usually 15). I wish I had a dollar for every time I've had similar corrections.


Quote: DogHand

Wizard,

Rule 5 has a typo on the payout for Big: it says 1.91, but on your EV table you have 1.95.

Dog Hand



Thanks, got it.
"For with much wisdom comes much sorrow." -- Ecclesiastes 1:18 (NIV)
Ace2
Ace2
  • Threads: 32
  • Posts: 2672
Joined: Oct 2, 2017
May 22nd, 2019 at 12:17:04 AM permalink
Quote: Wizard

I get the theory about finding the generating function, but to actually do it would be the equal amount of work as just cycling through all combin(80,20) balls by brute force.
.

Exactly. This is why I’ve never spent much time studying generating functions.

I prefer slick, formulaic solutions and would be much more impressed by an estimate accurate within .1% to an exact solution that was so voluminous that it required programming.

You can probably just use the expectation (810) and standard deviation (90) to get decent estimates for values within 2 or 3 SDs of expectation.
Last edited by: Ace2 on May 22, 2019
It’s all about making that GTA
ZPP
ZPP
  • Threads: 2
  • Posts: 31
Joined: Feb 7, 2010
May 22nd, 2019 at 1:17:58 AM permalink
Quote: Wizard

I didn't get very far with the code above. My C++ compiler didn't like this statement and I don't understand what it is supposed to:

typedef uint64_t num_t;

However, I can't ask anyone to teach me a new computer language. I was half finished doing it this way anyway, so will try to complete it with my own code.

Thanks ZPP for your help!

You're welcome.
The typedef defines num_t as uint64_t. The point is that (80 choose 20) is near the limit of 64-bit int, so if I want to switch to double, for larger values, I only have to change that one typedef (and the printfs). Also, if I wanted to use an old version of MSVC that doesn't support uint64_t, I would only have to change the one typedef (and the printfs) to usigned __int64... :)

Quote: Wizard

Damn Microsoft! Excel allows for only so many significant digits (usually 15).

I'm pretty sure Excel is just using "double" for everything internally.
Wizard
Administrator
Wizard
  • Threads: 1493
  • Posts: 26485
Joined: Oct 14, 2009
May 22nd, 2019 at 6:25:48 AM permalink
Somehow I think there is an elegant pattern to this problem somewhere. Some variant of Pascal's Triangle. However, if it were easy to find, I'm sure somebody would have by now.
"For with much wisdom comes much sorrow." -- Ecclesiastes 1:18 (NIV)
Wizard
Administrator
Wizard
  • Threads: 1493
  • Posts: 26485
Joined: Oct 14, 2009
May 22nd, 2019 at 7:05:53 PM permalink
Here is my new page on this topic, Keno Sums. I welcome all comments and corrections on it.

Again, thank you to all who contributed to this topic!
"For with much wisdom comes much sorrow." -- Ecclesiastes 1:18 (NIV)
rsactuary
rsactuary
  • Threads: 29
  • Posts: 2315
Joined: Sep 6, 2014
May 22nd, 2019 at 7:39:58 PM permalink
Thanks for your work on this... I didn't check through the math, but it seems weird that the return for exactly 810 is about half of every other bet.

ETA: You seem to have the number of combinations correct.. must be accurate... weird.
Wizard
Administrator
Wizard
  • Threads: 1493
  • Posts: 26485
Joined: Oct 14, 2009
May 22nd, 2019 at 9:44:10 PM permalink
Quote: rsactuary

Thanks for your work on this... I didn't check through the math, but it seems weird that the return for exactly 810 is about half of every other bet.



I figure that's because the payout is larger. More risk to the casino to offer it.
"For with much wisdom comes much sorrow." -- Ecclesiastes 1:18 (NIV)
Klopp
Klopp
  • Threads: 7
  • Posts: 16
Joined: Mar 10, 2020
March 11th, 2020 at 12:07:52 PM permalink
An insightful approximate solution can be given. The probability distribution of the sum of the 20 keno balls can be very closely approximated by a normal distribution with expected value 810 and standard deviation 97.2.
In chapter 9 of the book Surprises in Probability written by Henk Tijms a more general situation is considered: a lottery in which r different numbers are randomly drawn from the numbers 1 up to s. Then the sum of the drawn numbers has an approximate normal distribution with expected value r(s+1)/2 and as standard deviation the square root of r(s+1)(s-r)/12 provided that s is not too small . An insightful and useful result.
Last edited by: Klopp on Mar 11, 2020
  • Jump to: